<<

CHAPTER 16 Brad Frankum, Monique Wei Meng Lee and Jessie A Lee

–Sjögren’s syndrome (SS) CHAPTER OUTLINE –Infl ammatory myopathies –Anti-synthetase syndrome • KEY CONCEPTS IN IMMUNOBIOLOGY – and CREST syndrome – Innate and adaptive immunity –Mixed connective tissue (MCTD) –Specifi city and diversity –Antiphospholipid syndrome (APS) –Immunological memory – IgG4-related disease –, and immunodefi ciency • PRIMARY – Immunity, infl ammation and tissue repair – Large-vessel vasculitis –Understanding immunobiology –Medium-vessel vasculitis – Manipulation of the –Small-vessel vasculitis – Single- vasculitis • ALLERGIC DISEASE –Variable-vessel vasculitis – – Allergic (AR) and allergic • AUTOINFLAMMATORY DISORDERS conjunctivitis (AC) – Familial Mediterranean (FMF) –Chronic rhinosinusitis – TNF-receptor-associated periodic syndrome – Atopic (AD) (TRAPS) – Food –Urticaria and –Primary immunodefi ciency – Insect venom allergy –Secondary (acquired) immunodefi ciency • • HIV/AIDS –Hypereosinophilic syndrome (HES) –Epidemiology – Risk factors for HIV DISORDERS – Pathophysiology –Cutaneous mastocytosis (CM) – Clinical features and diagnosis –Systemic mastocytosis (SM) –Management –Prognosis • SYSTEMIC – Systemic erythematosus (SLE)

507 Essentials of internal

KEY CONCEPTS IN mechanisms such as coughing, sneezing, urination, and mucus production. IMMUNOBIOLOGY • The innate immune response is rapid, fi rst-line, and prevents tissue invasion in most instances. • Innate and adaptive immunity are unique but heavily interdependent entities. • Epithelial surfaces also provide chemical barriers to infection in the form of secreted antimicrobial , • Specifi city and diversity are the foundation for success- antibacterial and fatty acids, as well as microbio- ful adaptive immunity. logical barriers in the form of commensal microbiota that • Immunological memory is the basis for immunity and compete with pathogens. immunization. • Phagocytic cells can recognize a restricted number of • Hypersensitivity, autoimmunity and immunodefi ciency cell- surface molecules on microbial pathogens, and are the key drivers of immunopathology. be directly activated. These molecules are known as • Immunity, infl ammation and tissue repair are essential pathogen- associated molecular patterns (PAMPs). and interdependent components of a complex system to Examples of receptors that can recognize PAMPs include maintain health. Toll-like receptors, and mannose-binding lectin (MBL). • Understanding immunobiology is the key to logical • There are also receptors in the cytoplasm of cells that can diagnosis and treatment of infection. recognize microbial products. This is obviously important • Manipulation of the immune system is central to the for dealing with pathogens that have evaded defense mech- current, and future, treatment of a vast range of anisms in the extracellular environment. Examples include : allergy, autoimmunity, , infection NOD-like and RIG-1- like receptors and MDA-5. and transplantation. • The innate immune system also recognizes molecules released from damaged or infected cells. These mol- Innate and adaptive immunity ecules are known as DAMPs, or damage- associated Innate and adaptive immunity are unique but heavily inter- molecular patterns. dependent entities. Their key features are described in » DAMPs help to activate both the innate immune Table 16- 1. response and the infl ammatory response. • Complement is a set of plasma that form a cas- Innate immune response cade to ensure amplifi cation of both innate and adaptive Innate immunity refers to that section of the immune system immune responses. that does not require specifi c recognition to elimi- » Complement can be activated by the presence of nate a pathogen. It includes barrier functions such as integ- microbial cell surfaces, by the presence of immune rity of the and mucous membranes, and physiological complexes, or by the binding of MBL.

Table 16-1 Innate versus adaptive immunity

KEY FEATURES INNATE IMMUNE SYSTEM ADAPTIVE IMMUNE SYSTEM

Cells , -, , B and T lymphocytes, plasma cells basophils, mast cells, dendritic cells, natural killer cells

Receptors PRRs, Toll-like receptors B-cell receptors (immunoglobulins), T-cell receptors

Eff ectors Molecular—complement, acute phase reactants, Humoral—B cells, Cellular—phagocytes (neutrophils, monocyte- Cell-mediated—cytotoxic T cells macrophages), cells that produce infl ammatory mediators (eosinophils, basophils, mast cells), natural killer cells

Specifi city Absent Antigen-specifi c

Memory Absent (does not require prior contact) Antigen-specifi c (requires prior contact)

Response time Immediate Delayed

Response Not enhanced by prior contact Enhanced by prior contact magnitude

PRR, pattern-recognition receptor.

508 Chapter 16 Immunology

» Activated complement proteins have chemotactic actions to trigger phagocytosis, as well as eff ector Box 16-1 function, resulting in cell killing through the mem- The fi ve stages of the adaptive brane attack complex. immune response Activation of the innate immune response results in recruit- ment of the adaptive response. This is achieved predominantly 1 Antigen capture via antigen presentation by dendritic cells and macrophages to 2 Recognition of antigen lymphocytes, but is augmented by changes in milieu. 3 Activation of lymphocytes In turn, the adaptive immune response utilizes eff ector cells of the innate system. 4 Antigen elimination 5 Decline of immune response Adaptive immune response The three key components of the adaptive immune response are diversity, specifi city and memory. » are the cells which recognize self- antigen, so are • Diversity is manifested by the deliberate generation of antigen-specifi c, but develop in such a way as to many lymphocyte clones, but selecting only those that suppress response toward self- antigen rather than recognize foreign but not self- antigen for survival. initiate it • Specifi city refers to each lymphocyte clone having » function to maintain self- tolerance and suppress receptors of one specifi city, and responding to that infl ammatory responses. specifi c antigen only. The adaptive immune response proceeds through the fi ve • Memory is maintained by retention of antigen-specifi c stages listed in Box 16- 1. clones of both B and T lymphocytes, and the ongoing The major histocompatibility complex (MHC) is a set of production of specifi c antibody, in perpetuity. genes which encode for cell- surface proteins that represent a critical step in presenting foreign antigen to the eff ector cells A number of diff erent cells are involved (Figure 16- 1): of the immune system. • B cells diff erentiate into plasma cells to secrete antibody. • Cells which are able to stimulate CD8+ cytotoxic T • CD4+ T- helper cells direct other cells to perform cells must be able to process cytosolic antigen, and pres- eff ector functions. ent the antigen to the cell surface in conjunction with • CD8+ cytotoxic T cells, when activated, kill cells MHC class I . These are all nucleated cells. infected by intracellular organisms, e.g. viruses. • Cells which are able to present antigen to CD4+ T- helper • T- regulatory cells: cells, and stimulate them, must be able to process endo- » are a subset of CD4+ T cells which express high lev- cytosed antigen, and express MHC class II antigens els of CD25, and a transcription factor called FoxP3 in conjunction with the foreign antigen on the cell sur- » are generated centrally in the , or peripher- . These are dendritic cells, monocyte-macrophages ally in secondary lymphoid tissue and B lymphocytes, and are collectively referred to as antigen- presenting cells. Specifi city and diversity Specifi city and diversity are the foundation for successful Adaptive adaptive immunity. immune system • During development in the (B cells) and the thymus (T cells), millions of diff erent genetic recombinations occur in the genes that encode for lym- APC Lymphocytes phocyte receptors. • Immunological specifi city refers to the ability of the immune system to possess cells that are capable of rec- ognizing one specifi c antigen only. This is the function T cells B cells of the lymphocyte. CD3 CD19, CD20 » B lymphocytes express cell-surface receptor (immu- noglobulin) which is specifi c for one antigen. When secreted, this immunoglobulin can be arranged as a pentamer (IgM), dimer (IgA) or monomer (IgG, T-helper T cytotoxic T reg IgE, IgD), but still remains specifi c for one antigen CD3, CD4 CD3, CD8 only. » T lymphocytes express the T- cell receptor which is Figure 16-1 Cells of the adaptive immune system. also specifi c for one antigen only. APC, antigen-presenting cell; T reg = T-regulatory • Immunological diversity refers to the capacity of cell the immune system to have lymphocytes capable of

509 Essentials of

recognizing every conceivable antigen. The vastness of –Eff ector T lymphocytes are either cytotoxic this repertoire is staggering. T lymphocytes or T-helper cells. » In generating suffi cient diversity amongst the lym- –Eff ector B lymphocytes are plasma cells, which phocyte population, the immune system needs a secrete antibody. system in place during the requisite genetic rear- • Upon exposure to antigen, naïve lymphocytes trans- rangements that allows production of clones respon- form into lymphoblasts, which are larger, more meta- sive to foreign antigen, but disallows clones that are bolically active cells. Some of these lymphoblasts then self- reactive. diff erentiate into eff ector cells. » Failure to delete these autoreactive clones from being generated, or failure to recognize and deal • Cytotoxic T cells can be recognized by the cell- surface with those that escape, is the basis for autoimmunity. markers CD45, CD3 and CD8. » T-helper cells express CD45, CD3 and CD4. They produce cytokines that stimulate B lymphocytes Immunological memory and macrophages. Immunological memory is the basis for immunity and » Both cytotoxic and T- helper cells express molecules immunization. that draw them to sites of infl ammation, such as • Lymphocytes exist in three states: naïve, eff ector and adhesion molecules. memory (Table 16-2). • Activated (eff ector) B cells express IgG, IgA and IgE on » Naïve lymphocytes their surface predominantly. They have high expression – Naïve lymphocytes have not encountered spe- of CD27. cifi c antigen, and survive for several months • Aft er resolution of the eff ector response to specifi c anti- only if not exposed to antigen. There is a gen, a population of memory lymphocytes persists in steady-state replacement process for these the circulation. naïve cells. This may wane in older people » when immunosenescence occurs. These cells may survive for months or years. » – Naïve T lymphocytes can be recognized by the Like plasma cells, memory B cells express IgG, IgA cell- surface marker CD45RA. or IgE, but can be distinguished by being small with – Naïve B lymphocytes express IgM or IgD on less cytoplasm. Memory B lymphocytes can be dis- their surface and have low expression of the tinguished from naïve B cells by high expression of cell- surface marker CD 27. CD27. – Naïve lymphocytes circulate predominantly to » Like eff ector T cells, memory T lymphocytes lymph nodes, and have low expression of sur- express surface molecules that draw them to sites face molecules such as adhesion molecules that of infl ammation, such as adhesion molecules. They would draw them to sites of infl ammation. also predominantly express CD45RO. Memory » Eff ector lymphocytes T lymphocytes diff er from eff ector T cells, how- –Eff ector lymphocytes arise aft er specifi c anti- ever, by expressing high amounts of CD127. gen exposure. The unique qualities of memory cells make them far more –They have a short half- life. effi cient at responding to antigen challenge than naïve cells.

Table 16-2 Features of naïve, eff ector and memory lymphocytes

EFFECTOR LYMPHOCYTES (B CELLS = PLASMA CELLS; NAÏVE T CELLS = T-HELPER CELLS, MEMORY FEATURE LYMPHOCYTES CYTOTOXIC T CELLS) LYMPHOCYTES

Survival Several months Days Months to years

B cell surface IgM, IgD IgG, IgA, IgE, high CD27 IgG, IgA, IgE, high CD27 markers Low CD27

T cell surface CD45RA T-helper cells: CD4+, CD25, CD40 ligand, CD45RO markers CD45RO, low CD127 High CD127 Cytotoxic T cells: CD8+, CD25, CD45RO, low CD127

T cell adhesion Low High High molecule expression

Cytoplasm size Small Large Small

510 Chapter 16 Immunology

This forms the basis of adaptive immunity. It also explains defi ciencies, are more prone to SLE. When they incur the effi cacy of : tissue damage, e.g. with exposure to ultraviolet (UV) • For a given antigen, there are larger numbers of mem- radiation or through infection, they are then more likely ory than naïve lymphocytes. This proportion increases to suff er a disease fl are. The cell breakdown that occurs with age. from the environmental factor releases more autoantigen (in the case of SLE this is cellular nuclear material), • Memory T lymphocytes are more readily drawn to sites and the autoreactive cells then produce more antibody, of infection and infl ammation. which combines with the autoantigen. The resultant • Memory B lymphocytes have already class-switched to immune complexes then elicit an infl ammatory response express the higher-affi nity cell- surface immunoglobulins through combination with Fc receptors on phagocytes, —IgG, IgA and IgE. and via complement activation. • Memory lymphocytes respond to antigen stimulation Immunodefi ciency can be congenital or acquired. several days faster than do naïve lymphocytes. • Most congenital (primary) immunodefi ciency is genet- ically based. All components of the immune system Hypersensitivity, autoimmunity and can be thus aff ected, although severe forms of primary immunodefi ciency immunodefi ciency are rare. Hypersensitivity, autoimmunity and immunodefi ciency are • Acquired immunodefi ciency can result from environmen- the key drivers of immunopathology. tal factors such as infection (e.g. human immunodefi ciency • Immunological reactions that are inappropriate in inten- virus [HIV] infection), or malnutrition, or from aging, or sity, inappropriately targeted or inadequately regulated malignancy. Iatrogenic factors such as chemother- are labeled hypersensitivity disorders. apy or immunosuppressive drugs are also important. • There are four classic types of hypersensitivity (see Table 16- 3). Immunity, infl ammation and tissue Autoimmunity is the basis for a large amount of human repair disease. Immunity, infl ammation and tissue repair are essential and • Autoimmune disease may be organ-specifi c, as in interdependent components of a complex system to main- Hashimoto’s thyroiditis and pernicious anemia; or sys- tain health. temic, as in systemic (SLE). • Infection or tissue damage incites an infl ammatory • Autoimmunity results from a combination of genetic response. and environmental factors. For example, patients with • Immune system activation results in the release of a vari- certain human leukocyte antigen (HLA) subtypes, ety of cytokines (chemokines) that function to attract and with certain inherited complement component eff ector cells.

Table 16-3 Classifi cation of hypersensitivity

TYPE I II III IV

Mechanism Immediate Antibody- Immune-complex- T-cell-mediated mediated mediated

Immunopathology IgE-mediated Antibody directed Deposition Mediated by CD4+ T-helper cells Mast-cell at tissue antigens of immune of the TH1 or TH17 subsets, or by degranulation Complement complexes in cytotoxic T lymphocytes Production of activation vascular beds May be inappropriately directed prostaglandins Fc-receptor- Fixation of against self-antigens, or foreign and leukotrienes mediated complement antigens (e.g. nickel in jewelry infl ammation Fc-receptor- causing ) mediated There may be collateral damage infl ammation in responses against intracellular infection (e.g. viral hepatitis)

Disease examples Pernicious anemia Contact dermatitis Hashimoto’s thyroiditis Post-streptococcal Type I mellitus Anaphylaxis GN

GN, glomerulonephritis.

511 Essentials of internal medicine

• Other cytokines are infl ammatory, resulting in eff ects » treatments that block the release of mediators (mast- such as vasodilatation, increased vascular permeability, cell stabilizers) or the eff ect of mediators (antihista- fever and pain, all of which enhance the eff ectiveness of mines, leukotriene antagonists, ) the immune response in limiting damage caused by the » treatments that suppress the immune system to or infection. counteract the activity of B lymphocytes produc- • Clotting mechanisms are enhanced by infl ammatory ing IgE, or eff ector T lymphocytes (corticosteroids, cytokines. This both results in limitation of loss other immunosuppressives) from damaged tissue, and hinders travel of infection to » treatments that manipulate and ameliorate the aller- distant sites. gic response () » specifi c anti- IgE (). • Yet other cytokines trigger tissue repair in order to • Autoimmune disease will predominantly require restore tissues once the insult has been dealt with. to counteract the eff ect of patho- For the most part, these systems function smoothly to logical antibody production or autoreactive T cells. maintain homeostasis, and control mechanisms are in • Agents such as corticosteroids and have place to curtail responses when function is restored. When both anti- infl ammatory and immunosuppressive actions, components of the systems are defi cient, or control mech- depending on dose. anisms fail to limit an excessive infl ammatory response, disease results. • Targeted biological therapy, using monoclonal antibod- ies against cellular receptors or cytokines, is increasingly being used to treat both and autoimmune Understanding immunobiology disease (Table 16- 4). Understanding immunobiology is the key to logical diagno- • Although largely unhelpful for treating infection in sis and treatment of infection. immune- competent hosts, pooled human immuno- • The response of the immune system to dealing with a globulin can be very eff ective to modify immune variety of diff erent pathogens forms the basis of eff ective responses in the setting of autoimmunity (Box 16- 2). treatment. • Manipulation of the immune system has enabled the • Defi ciencies in components of the innate immune sys- use of tissue transplantation to treat a variety of life- tem, for example , will make the individual threatening diseases. especially susceptible to bacterial and fungal infection, » Organs can now be transplanted from donors who and thus or antifungal therapy should be con- are less immunologically matched to the recipient sidered promptly in the setting of early signs of infection. than ever before. • Antibody defi ciencies will similarly predispose to extra- cellular infection. Table 16-4 Biological agents with a role in • Cellular immune defi ciency predisposes to infection autoimmune disease with intracellular organisms such as viruses and myco- bacteria. The approach to an ill patient in this setting will BIOLOGICAL therefore diff er both diagnostically and therapeutically. AGENT ANTIBODY TARGET • At present the capacity to iatrogenically ‘boost’ the immune system is limited. Replacing immunoglobu- Infl iximab TNF-alpha lin in those who are defi cient is of proven benefi t, but the use of pooled donor immunoglobulin for infection otherwise is not. T- cell defi ciencies are even more prob- lematic, with a very small number of congenital, severe Certolizumab immunodefi ciencies being managed with bone marrow transplantation though with variable results. IL-1R • Rarely, the administration of cytokines for therapy is IL-6 used. An example is the success of - gamma for chronic granulomatous disease. CD20 (anti-) Ocreluzimab Manipulation of the immune system Manipulation of the immune system is central to the current, and future, treatment of a vast range of human disease: allergy, CD80 and CD86 autoimmunity, malignancy, infection, and transplantation. BAFF • While immunosuppression remains the mainstay of treatment for a large number of immunologically medi- alpha-4 ated diseases, newer and more sophisticated approaches, CD11a especially the growing use of biological agents, off er increasing hope for more targeted and less toxic therapy. BAFF, B-cell activating factor; IL, ; IL-1R, interleukin-1 receptor; TNF, tumor factor. • Strategies for dealing with allergic disease include:

512 Chapter 16 Immunology

Box 16-2 Evidence-based uses for high-dose immunoglobulin therapy in autoimmune disease Neurological disease Hematological disease Other autoimmune disease • Myasthenia gravis • Immune thrombocytopenic • CIDP (chronic infl ammatory • Immune neutropenia • Vasculitis (some forms, limited demyelinating polyneuropathy) • Immune hemolytic anemia evidence) • Guillain–Barré syndrome • Parvovirus B19-associated aplasia in • Dermatomyositis immunocompromised patients • Multifocal motor neuropathy • Lambert–Eaton syndrome • Stiff person syndrome

» So successful has been immunosuppression to sup- press tissue rejection, that the major risks to the trans- Box 16-3 planted patient lie in the susceptibility to infection Causes of anaphylaxis and the long-term threats of metabolic disease and malignancy, rather than from organ rejection per se. IgE-dependent Non-IgE-dependent • Similarly, the role of immunotherapy is likely to increase • Foods— and tree • Radiocontrast media in the treatment of malignancy in the future, both by nuts, eggs, crustaceans, • Drugs—non-steroidal activating immune mechanisms directed toward can- seeds, cow’s milk anti-infl ammatories, cer cells and by supporting the immune system in those • Drugs—beta-lactam opioids who have been immunosuppressed by the eff ects of che- , neuromuscular • Physical causes— motherapy, radiotherapy or targeted biological therapy. blocking drugs, local exercise, cold, heat anesthetic agents, insulin, • Highly tissue- damaging consequences can result from agents, • Idiopathic anaphylaxis the infl ammatory response induced by a variety of monoclonal , • Systemic . This is largely through the systemic eff ects of immunotherapy mastocytosis pro- infl ammatory cytokines. • Insect stings— » A limited role has been defi ned for the use of immu- Hymenoptera venom (, notherapy that counteracts this excessive infl am- yellowjacket/wasp, hornet) matory response, but it is likely that agents will be discovered in the future. • Latex, chlorhexidine » It is also possible that more specifi c and safer thera- • Inhalants rarely—horse pies will be developed in the future that can augment hair, cat hair components of the immune response to infection, enhancing eradication of pathogens while preserv- ing tissue integrity and function simultaneously. chemicals such as histamine, and rapid production of others such as leukotrienes and prostaglandins. • This can be triggered by IgE- dependent or non- IgE ALLERGIC DISEASE dependent mechanisms. • Non-IgE- dependent mechanisms may be IgG- or Anaphylaxis complement- driven, or a result of direct mast- cell Epidemiology activation. Anaphylaxis is a life-threatening disorder, with a lifetime Clinical features prevalence that is unknown but may be as high as 2%. The incidence is increasing. Children and young adults are • The majority of patients with anaphylaxis have cutane- aff ected more frequently. The diagnosis is oft en missed. ous involvement of the skin and mucous membranes, in Fatalities due to anaphylaxis are rare, but, again, may be the form of urticaria and angioedema. under- reported. Common causes are shown in Box 16- 3. • Generalized fl ushing can also occur. • Pruritus is usual. Pathophysiology • Respiratory involvement in the form of stridor, second- • Anaphylaxis results from widespread mast- cell and/or ary to laryngeal and upper airway , or wheezing, basophil degranulation, resulting in the immediate release secondary to bronchospasm, is necessary for diagnosis, of pre- formed vasoactive and smooth- muscle- reactive unless there is plus skin involvement.

513 Essentials of internal medicine

• Hypotension may result in syncope, confusion or • Patients on beta-adrenoceptor blockers are at increased incontinence. risk, due to antagonism of the eff ect of epinephrine. • Gastrointestinal involvement in the form of nau- • Lack of preparedness for community self-management sea, abdominal cramps, and/or is in those at risk, through the availability of and ability common. to appropriately use an self-injecting device, Diff erential diagnosis also confers a worse prognosis. • The diagnosis of anaphylaxis is clinical, but elevated serum tryptase in a sample taken within 3 hours of the CLINICAL PEARL event, or serum histamine taken within 1 hour, is usu- Anaphylaxis is a clinical syndrome, with no distinction ally confi rmatory. made between IgE-mediated and non-IgE-mediated • Other causes of shock need to be considered, especially causes. The term ‘anaphylactoid’ is no longer used. if there is no evident mucocutaneous involvement. • When anaphylaxis is suspected, it is important to attempt to ascertain the trigger through careful history Allergic rhinitis (AR) and allergic taking, and confi rmatory tests where relevant such as conjunctivitis (AC) skin-prick tests or serum-specifi c IgE for foods, drugs or venoms. Epidemiology Management • Allergic rhinitis and aff ect up to The key to the management of anaphylaxis is the early 30% of the population. administration of epinephrine (adrenaline), and fl uid resus- • They are more common in developed countries and citation. Fluid volume expansion may need to be aggressive temperate climates. to restore adequate circulation. • They frequently coexist in the same individual, but AR is more common than AC. It is uncommon to see AC CLINICAL PEARL in an individual without AR, but this can occur. The dose of epinephrine (adrenaline) to be used in • Both genders are aff ected equally. anaphylaxis should be 0.01 mg/kg, up to a maximum dose of 0.5 mg. This should be given intramuscularly • AR and AC form part of the atopic phenotype, with (1:1000 dilution) or intravenously (1:10,000 dilution). aff ected individuals more likely to suff er from asthma, Intravenous epinephrine administration should only occur in the setting of cardiac monitoring. Doses can food allergy and/or . be repeated as needed if response is poor. • In particular, asthma is found in 15–38% of patients with AR, and nasal symptoms are present in up to 85%

• H1 and corticosteroids are useful adjuncts of patients with asthma. AR is a risk factor for asthma, in treatment, but should never be used as an alternative and uncontrolled moderate-to-severe AR aff ects asthma to epinephrine (adrenaline). control. • Any patient with anaphylaxis should be observed in • Onset is typically in childhood aft er the age of about hospital for at least 4 hours, and up to 24 hours in severe 5 years until young adulthood, but can come on later in cases. life, especially with a change in geographical location. • Long term, patients must be instructed to strictly avoid identifi ed triggers, and subsequent challenge to assess for ongoing clinical risk should only ever be undertaken Pathophysiology in a hospital setting under the supervision of appropriate Both AR and AC result from allergic sensitization to inhal- specialists. ant allergens. • is only used in the setting of ana- • Nasal mucosal and/or conjunctival mast cells have surface phylaxis for Hymenoptera venom allergy, and more recently IgE that is specifi c for allergen in aff ected individuals. for single food allergy with the use of oral immunotherapy for allergy. • Combination of allergen with IgE bound to the mast- cell surface aft er inhalation into the nose, or settling on Prognosis the conjunctival surface, results in cross-linking of IgE and triggering of the allergic reaction locally. • Death is unusual in anaphylaxis, but is probably under-recognized. • Common allergens implicated in AR are the house dust • Prognosis is worse in patients with coexisting asthma, mite, grass and tree , and animal danders (see other respiratory disorders, and underlying cardiovascu- Box 16- 4). Suff erers may be mono-sensitized, but more lar disease. commonly react to multiple allergens.

514 Chapter 16 Immunology

Diff erential diagnosis Box 16-4 Diagnosis is usually not diffi cult, and can be confi rmed by Common airborne allergens the demonstration of specifi c IgE to an appropriate allergen, either by skin- prick testing (Figure 16-2) or by the detection Perennial allergens Seasonal allergens of specifi c IgE in serum by various techniques. • House dust mite— • Grass pollen—e.g. Dermatophagoides perennial ryegrass, • Non- allergic rhinitis can cause similar symptoms to pteronyssinus, D. farinae Bermuda grass, AR, in the absence of demonstrable allergic sensitiza- tion. The etiology is unknown. • Animal danders—dog, » cat (Fel d 1), horse hair • Tree pollen—e.g. , Vasomotor rhinitis is a subset of non-allergic rhi- pine nitis, and is characterized by nasal congestion and • Cockroach—German rhinorrhea that is provoked by non- specifi c envi- and American species ronmental stimuli such as rapid temperature change • Molds—e.g. Alternaria and strong odors. tenuis, Cladosporium » NARES is non- allergic rhinitis with eosinophilia, herbarum where excess eosinophils are found on a nasal smear. This is presumed to be due to local mucosal aller- gic sensitization where the culprit allergen/s is not demonstrable systemically. CLINICAL PEARLS • Chronic rhinosinusitis is more likely if the individual has thick or purulent nasal discharge, sinus and/or den- • The demonstration of a specifi c IgE to an allergen in tal pain, post- nasal drip and/or hyposmia or anosmia. an individual confi rms sensitization to that allergen rather than clinical allergy. • Nasal congestion can occur in the latter stages of preg- • Many individuals are sensitized to allergens that nancy, and is known as rhinitis. cause no clinical symptoms. • Inappropriate long- term use of nasal • Both in vivo (skin-prick tests) and in vitro (serum- sprays can cause rebound nasal mucosal congestion, and specifi c IgE testing) testing are highly sensitive for is known as rhinitis medicamentosa. the detection of allergic sensitization. Skin-prick testing, in expert hands, is more sensitive and spe- cifi c, and allows clearer identifi cation of individual Management allergens, whereas specifi c IgE tests can be used Management of AR and AC is outlined in Box 16-5, when patients have skin disease that renders skin overleaf. testing diffi cult, or when the patient is taking anti- histamines, which will give false-negative skin-prick Prognosis testing. • AR and AC have been shown to adversely aff ect quality of life when not treated, or treated sub- optimally. Clinical features • Symptoms generally persist for decades in the absence of allergen- specifi c immunotherapy, only abating in most • AR is characterized by nasal congestion and blockage, cases in middle age. rhinorrhea (usually watery), excessive sneezing, and nasal itch, in varying combinations. The local allergic reaction can also result in palatal itch, or itch within the ear. • AC results in redness, excessive tearing, and itching of the eyes. • Suff erers of AR and AC frequently complain of fatigue and irritability. • In AR, physical examination reveals pale, swollen infe- rior nasal turbinates. There may be partial to complete nasal occlusion. • In AC, conjunctivae may be injected, with variable chemosis. • In general, symptoms will be perennial when due to dust mite or animal danders, or seasonal with grass and tree pollen. This, however, may vary with climatic con- ditions and geography. Figure 16-2 Positive skin-prick tests • ‘Hayfever’ is a historical term that refers to the intense From Yu M-C et al. Allergic colitis in infants related to cow’s milk: rhinoconjunctivitis that occurs in the springtime in clinical characteristics, pathologic changes and immunologic individuals with pollen sensitization. fi ndings. Pediatr Neonatol 2013;54(1):49–55.

515 Essentials of internal medicine

Box 16-5 Management of allergic rhinitis and allergic conjunctivitis

Allergen avoidance • Loratadine • May be useful for house dust mite and animal danders, • Desloratadine although it is diffi cult to achieve • Safe. More eff ective for itching and rhinorrhea; less • Not possible for grass and tree eff ect on nasal blockage • Of limited overall effi cacy • Useful adjunct when used with other modalities

Intranasal corticosteroids Topical ocular therapy • • Lubricants • propionate • Antihistamines • • Mast-cell stabilizers • • Multiple-action drugs • • Corticosteroids • • Need to be used multiple times per day. Corticosteroids • Beclomethasone are not safe for long-term use (cataracts, infection) • Once daily; safe for long-term use • Adherence is often poor • Adherence often limits effi cacy • Costly

Intranasal antihistamines Allergen-specifi c immunotherapy • • Subcutaneous or sublingual: single or multiple allergens, with less evidence for effi cacy for multiple allergens • • Oral tablet immunotherapy is now also available for • Equally eff ective alternative to oral antihistamines but house dust mite and grass pollen faster onset of action • Risk of anaphylaxis with subcutaneous immunotherapy, Intranasal anti-muscarinic therefore requires expert administration and specialist supervision • Ipratropium • Off ers potential for long-term disease modifi cation • Rapid onset and prolonged action • Costly, but cost-eff ective in the long term • Useful adjunct if there is marked rhinorrhea

Systemic H1 antihistamines • Cetirizine • Fexofenadine

Chronic rhinosinusitis This contrasts with acute bacterial sinusitis, for which anti- biotics are generally effi cacious. Epidemiology Other risk factors for CRS include: • Chronic rhinosinusitis (CRS) is classifi ed as CRS with- • ciliary dyskinesia out nasal polyps or CRS with nasal polyps. • cystic fi brosis • The condition causes considerable morbidity. • sensitivity • Prevalence data suggest that CRS is common, perhaps • asthma. aff ecting up to 5% of the population. The type without nasal polyps is more common. Clinical features Chronic rhinosinusitis is generally subacute in onset, pre- Pathophysiology senting with: The etiology of CRS is unknown. It occurs more frequently • persistent nasal blockage (oft en with nocturnal snoring) in atopic than non-atopic individuals, suggesting a role for • facial pain and frontal headache allergic sensitization in some cases. • post- nasal drip. Humoral immunodefi ciency is a risk factor for CRS, which raises suspicion for the role of infection, as both bacte- Other symptoms may include: ria and fungi can be isolated in many cases. Treatment with • dental pain antibiotics and antifungal agents is, however, disappointing. • hyposmia or anosmia

516 Chapter 16 Immunology

• ear pain, fullness, or blockage Box 16-6 • persistent , with or without expectoration. Physical examination may be unremarkable, or reveal: Causes of nasal blockage • tenderness over the sinuses • Nasal mucosal conditions • mucopurulent secretions in the nasal passages or » Allergic rhinitis » Non-allergic rhinitis • nasal obstruction. » Infective rhinitis Diff erential diagnosis » Chronic rhinosinusitis (with or without nasal polyps) • The diagnosis of CRS can be confi rmed with com- • Adenoidal hypertrophy puted tomography (CT) or magnetic resonance imaging • Vasculitic conditions (MRI) of the paranasal sinuses, and/or direct visualiza- » ANCA-positive vasculitis tion with nasendoscopy. • Granulomatous disorders • Both persistent allergic rhinitis and non-allergic rhinitis » are less likely to cause facial pain, headache, post-nasal • Mass lesions drip, and alteration in the sense of smell than CRS. » Tumors of the nasal cavity, paranasal sinuses, or • ANCA-positive vasculitis should always be considered nasopharynx in a patient with chronic sinus-related symptomatology, • Anatomical obstruction especially if there is associated epistaxis, lower respiratory » Nasal septal deviation tract symptoms, or the patient is systemically unwell. » Trauma » Foreign bodies • Paranasal sinus tumors are very rare, but should be obvi- ous on imaging. • Some patients are prone to recurrent bouts of acute infective sinusitis. The patient should be symptom- Management free between episodes. The majority of cases of acute • CRS tends to be very - responsive, and infective sinusitis are, in fact, viral in origin, and resolve oft en a short course of moderate-dose without specifi c treatment. will relieve symptoms and result in signifi cant shrink- • If nasal ulceration or a saddle- nose deformity is found, age of polyp size. Eff ects are, however, temporary and the following conditions should be considered: patients can become inappropriately reliant upon ongo- » granulomatosis with polyangiitis (Wegener’s ing steroid . granulomatosis) • Intranasal steroids may benefi t a minority of patients, if » midline granuloma (non-caseating granuloma used continuously. without vasculitis) » tumor—carcinoma, lymphoma • Daily nasal lavage with solutions can help alleviate » relapsing polychondritis symptoms. » infection—, , , histo- • Patients with severe and ongoing disease will oft en plasmosis require surgical intervention. While oft en eff ective in » cocaine sniffi ng. the short term, disease will oft en recur aft er . Causes of nasal blockage are listed in Box 16- 6. • Patients with concurrent allergic rhinitis should be treated for that problem. CLINICAL PEARL • Leukotriene antagonists reduce symptoms in a minority Samter’s triad (aspirin-exacerbated respiratory disease, of patients. AERD) consists of sinonasal polyposis, aspirin hyper- • Antibiotic therapy should be used for acute fl ares of the sensitivity and asthma. It should be remembered that condition. Intranasal antibiotics used long- term are of approximately 10% of all asthmatics will have aspi- limited benefi t. rin-exacerbated asthma, but a much smaller number have Samter’s triad. It is extremely important to inquire • The monoclonal antibodies omalizumab, about aspirin or non-steroidal anti-infl ammatory drug and have all been shown to improve chronic (NSAID)-related symptoms in asthmatics, because ana- rhinosinusitis with nasal polyps, although availability is phylaxis may occur with inadvertent use. variable for this indication, largely due to high cost. Aspirin ‘desensitization’ (the graduated introduction of increasing doses of aspirin, commencing with min- Prognosis ute quantities) can be an eff ective treatment in Sam- CRS is a chronic disease that requires ongoing management. ter’s triad, but is dangerous due to the risk of severe asthma or anaphylaxis and should only be performed • Few patients are symptom- free at 5 years. under strict specialist guidance. Those patients who do • Patients should be monitored for the development of tolerate the regimen often then have gastrointestinal obstructive sleep apnea. side-eff ects from the high daily doses of aspirin that are required for maintenance of eff ective desensitization. • Frequent or long- term corticosteroid use will require monitoring for the usual complications.

517 Essentials of internal medicine

Atopic dermatitis (AD) A Atopic dermatitis is a common, chronic, pruritic skin con- dition which has signifi cant adverse eff ects on quality of life. The term ‘eczema’ is oft en used synonymously, but is less precise and is best avoided.

Epidemiology • Atopic dermatitis is estimated to aff ect more than 10% of young children. The prevalence decreases with age. • AD is more common in Western industrialized nations. • The majority of patients have the atopic phenotype, with signifi cant risk of concurrent or future food allergy, asthma and allergic rhinoconjunctivitis. B

Pathophysiology The development of AD results from a complex interplay of , skin barrier dysfunction and micro- biome imbalance, together with environmental insults. While it is clear that an abnormal Th2 response in the skin in AD leads to the overproduction of Th2 cytokines (see Table 16-5), which then drive excessive IgE production in response to a variety of food and environmental allergens, it is oft en incorrectly assumed that AD results solely from exposure to these allergens. Patients and parents of aff ected Figure 16-3 Typical patterns of atopic dermatitis in children oft en despair when the strict avoidance of allergens (A) an infant and (B) an adult to which they or their children are sensitized fails to ade- From: (A) Weston WL, Lane AT and Morelli JG. Color textbook quately control the disease. of pediatric , 4th ed. Elsevier, 2007. (B) Dr Harout Tanielian/Science Photo Library. Table 16-5 TH2 cytokines in AD

CYTOKINE ACTION IN AD

IL4/IL13 • Epidermal barrier dysfunction • Infl ammatory cell migration into AD lesions CLINICAL PEARL • Innate immune suppression Detection of allergic sensitization by demonstration of • Pruritus specifi c IgE by skin-prick or serum-specifi c IgE testing remains important in atopic dermatitis (AD), but results IL5 • Maturation, activation and migration must be interpreted in the context of the individual of eosinophils patient. • A child whose AD is under good control with topi- IL31 • Binds to receptors on sensory neurons cal treatment, and is tolerating foods such as and may directly induce pruritus or milk with no fl are in their disease after consuming these foods—even if they have demonstrable spe- Other components of this complex, multifactorial dis- cifi c IgE to these foods—should not be advised to ease include: remove wheat and milk from their diet. • A child who has demonstrable specifi c IgE to the • Skin dryness contributes toward poor barrier function. house dust mite may gain some benefi t from dust Filaggrin is a structural skin important for phys- mite reduction methods in the home, but the AD is ical integrity and prevention of water loss. Patients with very unlikely to be driven by this alone, so common loss of function in the Filaggrin gene suff er sense should prevail as to the extent to which aller- from a particularly severe form of AD. gen avoidance measures are pursued. • Chronic cutaneous bacterial colonization with Staph- Studies have shown that the presence of AD in children ylococcus and fungal colonization with Malassezia are predisposes to subsequent food allergen sensitization and known to contribute to skin fl ares and severe AD. sensitization and is the fi rst step in the atopic • Pruritus, resulting in persistent scratching and further march toward the development of food allergy, allergic rhi- damage to barrier function, and risk of infection. nitis and asthma.

518 Chapter 16 Immunology

Clinical features » Mild disease, and facial AD, can be safely treated The rash of AD: with topical 1% hydrocortisone. • is characterized by scaly, erythematous patches • 1% cream can be safely used for facial dermatitis, including eyelid dermatitis. • is very pruritic cream may be eff ective in severe cases. • may be discoid • Wet dressings or bandages used for several hours per day • occurs on very dry skin. can be highly eff ective for moisturization. The pattern of rash in AD (Figure 16-3) typically evolves with Patients who fail to improve despite adequate topical therapy increasing age, although at any point it may be generalized: should be considered for systemic therapy. The following • Infants—cheeks, torso, nappy area agents have been shown to be eff ective: • cyclosporine () • Toddlers—cheeks, perioral, fl exures, nappy area • • Children—eyelids, behind ears, fl exures, torso • methotrexate. • Adults—fl exures, limbs, hands, face, back. Several biologic agents have been proven eff ective in the Secondary infection is common, and may be manifested by treatment of AD, while a number of others are under inves- pustules, weeping, or worsening . tigation. See Table 16-6. Where possible, oral corticoster- oids should be avoided in the long-term management of Diff erential diagnosis AD. Rebound fl aring is usual when oral steroids are with- Pruritic, erythematous skin rashes are common. AD is best drawn, and it is diffi cult to justify the long-term side-eff ects distinguished by the age of onset and the pattern of rash, but of steroids in all but the most disabling cases of AD. may be mistaken for the following: Treatment of superimposed infection is important. • Allergic contact dermatitis, common causes of which • Short courses of anti-staphylococcal antibiotics should are: be used for obvious infection, or with disease fl ares. » nickel sulfate (jewelry) • Long-term low-dose antibiotics, e.g. once-daily co- » potassium dichromate (cement, leathers, paint) trimoxazole, may be of benefi t in some patients. » paraphenylenediamine (hair dyes, cosmetics) • Intermittent application of intranasal may » para- aminobenzoic acid (sunscreen) reduce staphylococcal carriage. » formaldehyde (cosmetics, shampoos) • Irritant contact dermatitis Prognosis • Psoriasis • Many aff ected children improve with age. • Scabies • Adult AD can be severe, and relatively resistant to therapy. • Drug eruptions. • An explanation for the exacerbations and remissions seen in the disease over time is not always obvious, and these Biopsy of the rash is rarely necessary, with the majority of may not be only on the basis of varying allergen exposure. patients displaying other atopic conditions. Food allergy CLINICAL PEARL Adverse reactions to foods can be IgE- or non- IgE- mediated, In a patient with a generalized erythrodermic dermato- due to immune or non-immune mechanisms. The term sis, a markedly elevated total serum IgE is highly sugges- ‘allergy’ should be reserved for immune-mediated reactions. tive of atopic dermatitis. Epidemiology • An epidemic of food allergy is being observed in the Treatment developed world. The incidence of food allergy is increas- Topical treatment is the mainstay of successful manage- ing sharply, and levels of up to 10% are being observed in ment of AD. preschool- aged children in the developed world. • Minimum twice- daily application of moisturizers and/or • Prevalence is much higher in children; not all children emollients. ‘grow out of ’ their food , so prevalence in adults • Adequate cleansing of the skin, while avoiding drying will increase over the next decades. soaps. • Adults can develop food allergy de novo. • Sterilization of the skin with dilute bleach baths 2–3 times per week can be useful. Pathophysiology • Liberal application of topical corticosteroids. • Non- IgE- mediated food reactions tend to mainly cause » AD that is worse than mild should be treated with gastrointestinal symptoms, such as bloating, cramping, moderate- to high-potency steroid creams or oint- and diarrhea. ments (e.g. mometasone, methylprednisolone) for • This may be chemically or immunologically mediated as long as it takes to get the disease under control. through non- IgE mechanisms, e.g. celiac disease.

519 Essentials of internal medicine

Table 16-6 Biological agents in AD

BIOLOGICAL AGENT MECHANISM OF ACTION EFFECTIVENESS IN AD Dupilumab Anti-IL4 receptor alpha. Approved for AD treatment. Signifi cantly improved AD severity, pruritus, quality of life, sleep.

Lebrikizumab Anti-IL13. Signifi cantly improved AD severity, improved skin clearance, quality of life, pruritus. Phase IIb.

Nemolizumab Anti-IL-31 receptor alpha. Signifi cant reduction in pruritus and AD severity. Phase II.

Baricitinib JAK 1/2 inhibitor. Signifi cantly improved AD severity, pruritus and sleep. JAK 1 inhibitor. Phase II.

Crisaborole Inhibitor of Approved for mild to moderate AD for adults and phosphodiesterase-4. children 2 years and older.

Ustekinumab Anti-IL-12/IL23 p40. May provide some benefi t for AD; more investigation needed.

Box 16-7 Common causes of food allergy Infants Toddlers Children Adults Cow’s milk Hen’s eggs Hen’s eggs Peanuts Soya bean Cow’s milk Peanuts Tree nuts Hen’s eggs Peanuts Tree nuts Seeds, e.g. sesame Tree nuts Seeds, e.g. sesame Crustaceans Fish and some Crustaceans Fruits and some vegetables

• Systemic reactions resembling anaphylaxis, or compo- • It is oft en caused by ingestion of fruits and some raw nents of anaphylaxis such as generalized urticaria, can vegetables, and is more common in those who are sen- occur through non- IgE mechanisms and can be danger- sitized to tree and grass pollen and suff er from seasonal ous. These can be due to a wide variety of food compo- allergic rhinitis. nents. Examples may include preservatives such as sulfi tes, • Reactions may be more common, and more severe, in or food-coloring agents. In these cases, it is presumed the pollen season. that release of histamine and other vasoactive molecules through non- IgE- mediated mechanisms is responsible. • Cooking will oft en denature the allergenic compo- nents of fruits and vegetables in OAS, and may allow • It is increasingly recognized that food components consumption. exacerbate symptoms of in signifi cant numbers of suff erers. Systemic food reactions • Atopic dermatitis can be exacerbated by both IgE and • Angioedema of the , face and upper airway can result non- IgE food reactions. from ingestion of culprit foods within minutes. • Allergy to a very wide variety of foods has been demon- • Gastrointestinal reactions of nausea, vomiting and diar- strated in individuals, but allergy to meats, cereals and rhea are common. vegetables are rare. Common causes of food allergy are listed in Box 16- 7. • Urticaria can be generalized. • Anaphylaxis with circulatory collapse and/or airway Clinical features obstruction can result in death.

Oral allergy syndrome (OAS) Diff erential diagnosis • results in oral and pharyngeal • When symptoms occur immediately aft er food inges- tingling, itching, and sometimes swelling. tion, it is usually obvious that the reaction is due to a • OAS uncommonly results in systemic symptoms or specifi c food, but young children may ingest culprit airway obstruction. foods when their carers are not watching them.

520 Chapter 16 Immunology

• Food allergy almost always occurs rapidly. • Non-IgE- mediated adverse food reactions may occur some hours aft er ingestion, and may result from com- ponents of several foods rather than an individual one. • Patients frequently attribute a wide range of symptoms to food. The needs to take a careful history to see if this is likely. • Testing for specifi c IgE with skin- prick tests or serum- specifi c IgE is generally sensitive for detecting food- allergen sensitization. For skin-prick testing, fresh fruits, vegetables, seafoods and meats are more reliable than commercially available extracts.

CLINICAL PEARL The size of the positive skin-prick reaction to a food, or the absolute level of serum-specifi c IgE to that food, is a guide to that person’s risk of an allergic reaction occurring with ingestion but not the severity of the reaction. The result must be used in conjunction with Figure 16-4 Patient with generalized urticaria the history and in comparison with the size of previous From Callen JP and Jorrizo JL, eds. Dermatological signs of internal reactions, or levels, to assess an individual’s risk. disease, 4th ed. Elsevier, 2009.

Treatment • Strict avoidance of culprit foods is essential. • Emergency action plans should be in place for all food- allergy suff erers. • Epinephrine self-injecting devices must be carried at all times by patients with food allergy who are assessed to be at risk of anaphylaxis. • Determining the level of risk is oft en diffi cult, and requires specialist assessment. • Food challenge testing is appropriate in individuals when there is evidence of developing tolerance, usu- ally evidenced by reduction in skin- prick reactivity or a lower specifi c IgE value, but must be conducted in spe- cialist clinics under strict protocols. • Introduction of common allergenic foods to infants Figure 16-5 Patient with angioedema should not be delayed, and all infants should be given From James WD, Berger T and Elston D. Andrew’s Diseases of the allergenic solid foods in the fi rst year of life, including skin: clinical dermatology, 11th ed. Elsevier, 2011. those at high risk of allergy. These foods include cow’s milk products, peanut butter, cooked egg and wheat products. • Acute urticaria or angioedema refers to short-lived and • In particular, early introduction of peanut in the fi rst self- limited episodes of disease. Causes are listed in year of life considerably reduces the risk of peanut Box 16-8. allergy in many children. • Chronic urticaria or angioedema refers to episodes • Immunotherapy (oral, sublingual and epicutaneous) for occurring daily, or almost daily, for a minimum period the treatment of food allergy is under and of 6 weeks. remains experimental. • At times the defi nition of the condition as either acute or chronic can be problematic when frequent or recur- Urticaria and angioedema rent acute episodes are occurring, but it is important to Urticaria (Figure 16-4) refers to raised, erythematous, make this judgment, given the diff ering pathophysiol- typically pruritic lesions that last for less than 24 hours. ogy and management approaches of the two conditions. Angioedema (Figure 16-5) is non-pitting swelling that typ- • Around 50% of patients will have both urticaria and ically involves mucous membranes but can also aff ect the angioedema simultaneously or successively, whereas skin, with a predilection for face, hands and feet, and genita- 40% will have urticaria alone, and 10% exclusively lia. Angioedema can aff ect the gut. angioedema.

521 Essentials of internal medicine

Diff erential diagnosis Box 16-8 • Other pruritic skin conditions can occasionally urti- Causes of acute urticaria and cate, especially aft er extensive scratching, e.g. atopic angioedema dermatitis. • Cellulitis is sometimes mistaken for angioedema, but IgE-mediated Non-IgE-mediated is generally painful and associated with systemic symp- Food allergy Adverse food reactions toms such as fever. Insect sting allergy Drug reactions, e.g. aspirin sensitivity • Serum IgE is oft en moderately elevated in the setting Drug allergy of chronic urticaria. This should not be interpreted as Aeroallergen allergy, Radiocontrast media indicating an ‘allergic’ cause. e.g. horse hair Physical factors—cold, heat, exercise, sun • Approximately 10% of those with chronic urticaria will have demonstrable thyroid autoantibodies. Actual Inherited angioedema thyroid dysfunction is much less common, but these Idiopathic patients should remain under surveillance prospectively Infection, especially in children for the development of hypothyroidism.

CLINICAL PEARL Urticarial lesions that are painful or burn, last longer than 24 hours, or leave a bruise or stain, could be due CLINICAL PEARLS to urticarial vasculitis. A skin biopsy is indicated in this circumstance. • The patient who suff ers episodes of angioedema without any history of urticaria should be assessed for hereditary or acquired angioedema due to com- (HAE) plement component defi ciency or dysfunction. It is Hereditary angioedema is a rare but important cause of inappropriate to do this if urticaria is present. recurrent bouts of angioedema occurring in approximately • Angioedema without urticaria can also be the result of treatment with angiotensin-converting inhibi- 1 in 50,000 individuals. Urticaria is absent in this condition. tors and gliptins (dipeptidyl peptidase-4 inhibitors). • It is an autosomal dominant disorder that generally manifests in early childhood. • Episodes frequently aff ect the airway, and can be life- Epidemiology threatening. Intestinal involvement is also frequent, caus- • It is estimated that approximately 20% of individu- ing severe and frequent misdiagnosis. als will have an episode of acute urticaria during their • Episodes are oft en precipitated by events such as minor life. trauma, surgery or dental work. • Chronic urticaria is less common, with an estimated • It is due to a genetic defect in the , prevalence of 0.5–5%. specifi cally in the absolute quantity of C1-esterase inhibitor (type 1 HAE) or in its function (type II HAE). Pathophysiology • The primary mediator of swelling is bradykinin, which • Various environmental and food allergens can also cause predominantly acts on the bradykinin B2 receptor to contact urticaria. This usually remains confi ned to the area induce increased vascular permeability. Bradykinin is of skin or that the allergen directly rapidly metabolised by endogenous metalloproteases contacts, but can occasionally generalize. An example including angiotensin converting enzyme might be a cutaneous reaction to a plant experienced by • Diagnosis is made by demonstrating low C4 levels, and a gardener. low C1 inhibitor levels or function. • Approximately 50% of cases of chronic urticaria and • On-demand treatment of all attacks as early as possible is angioedema are autoimmune. recommended and all patients should carry on-demand • Autoimmune cases of chronic urticaria and angioedema at all times suffi cient to treat two attacks. are characterized by IgG antibodies directed against • Attacks should be treated with C1 inhibitor concentrate, autologous IgE, demonstrable by a positive wheal and icatibant, which is a selective bradykinin B2 receptor fl are to autologous serum on skin- prick testing. antagonist or ecallantide, a kallikrein inhibitor. • refers to urticaria and angioedema pro- • Pre-procedural or short-term prophylaxis should be voked by physical factors (Table 16- 7, overleaf ). administered prior to surgery, childbirth or dental sur- Regardless of the cause, the manifestations of the disease gery in the form of C1 inhibitor concentrate, as close as are the result of the release of histamine and other vaso- possible to the start of the procedure. active factors from mast cells, resulting in vasodilatation and • All patients should be assessed for long-term prophy- increased vessel permeability. laxis with C1 inhibitor or androgens.

522 Chapter 16 Immunology

Table 16-7 Physical urticaria

PHYSICAL FACTOR CLINICAL FEATURES

Dermographism Common condition Wheals only appear after scratching Benign, usually self-limited condition

Heat = Small, transient directly after skin becomes hot, e.g. showering Not associated with angioedema

Cold = cold-induced urticaria Generally aff ects exposed skin Can produce angioedema Anaphylaxis is a risk, e.g. with immersion in cold water

Exercise = exercise-induced Often only occurs after moderate to strenuous exercising urticaria Anaphylaxis can result

Water = Occurs with any water contact on skin, regardless of temperature Generally does not aff ect mucous membranes, e.g. with drinking.

Sun exposure = Presumed to be due to UV radiation

Vibration = vibratory urticaria Occurs particularly with repetitive stimuli, e.g. using a jackhammer Angioedema is frequent

Delayed-pressure = delayed- Frequently produces angioedema Occurs on skin subject to prolonged pressure, e.g. under tight clothes Often -resistant, and can be diffi cult to control

Food-dependent, exercise- Occurs after exercise, but only if specifi c food is consumed immediately beforehand induced Especially occurs with wheat and seafood Can cause anaphylaxis

• C1- esterase inhibitor defi ciency can be acquired, in • Cyclosporine may be used as fourth-line therapy if there about 15% of cases. This is generally in the setting has been no response to 6 months of treatment with of hematological malignancy and patients oft en have omalizumab. depressed C1q levels. • Daily prophylactic antihistamine therapy should be used for extended periods in the setting of chronic urticaria Treatment of urticaria and angioedema and angioedema. Many patients make the mistake of Acute episodes of urticaria and angioedema with an iden- relying upon p.r.n. (as required) treatment in this set- tifi ed cause should, as much as possible, be managed long- ting, with disappointing results. term with avoidance of identifi ed culprits. • Prophylactic H1 antihistamines may alleviate or reduce the severity of physical urticaria (e.g. taken prior to Antihistamine therapy exercise or cold exposure). • Adequate doses of second-generation, non-sedating H 1 • Addition of H2 antihistamines such as ranitidine or antihistamines are the mainstay of treatment for both famotidine may have a synergistic eff ect with H1 anti- acute and chronic urticaria, and angioedema. histamines, but are ineff ective by themselves. • The superiority of newer, non-sedating agents such as fex- ofenadine, cetirizine, loratadine, and desloratadine over the Alternative treatments older, sedating agents is well established. A standard dose • Acute severe urticaria or angioedema may occasionally of one of these second-generation H1 antihistamines is the require a dose of subcutaneous epinephrine. initial treatment. Patients who do not achieve adequate symptom control at 2–4 weeks should increase to up to 4 • Leukotriene antagonists such as can be times the usual recommended dose of antihistamines. tried for cases unresponsive to antihistamine. • Patients with CSU who have not responded to a 2–4 week • Short courses of corticosteroids can alleviate acute severe trial of 4 times the standard dose of a second-generation episodes. antihistamine should be considered for omalizumab, a • Long-term corticosteroids should be avoided for chronic against IgE, as add-on therapy. cases, unless all other pharmacotherapy has failed.

523 Essentials of internal medicine

• Azathioprine, mycophenolate, cyclosporine and metho- • Type I reactions may result in urticaria, angioedema, trexate should be used only under specialist supervision upper airway obstruction, bronchoconstriction, vomit- and monitoring. ing and diarrhea, or circulatory collapse. Prognosis • Type III reactions oft en manifest with fever, malaise, arthritis, maculopapular or vasculitic rash. Urinalysis Approximately 20% of chronic urticaria patients will have may be abnormal due to glomerulonephritis. ongoing symptoms aft er 5 years. • Cutaneous abnormalities due to drug reactions are Drug allergy numerous, and include maculopapular rash, vasculitis, urticaria, photosensitivity, erythema, erythema multi- Adverse reactions to drugs are extremely common, and may forme, cutaneous lupus, and fi xed eruptions. These may be predictably dose- related or idiosyncratic. The term ‘drug result from all types of hypersensitivity. allergy’ should be reserved for reactions that are likely to be Table 16-8 outlines specifi c serious drug reaction syndromes. immunologically mediated. • IgE- mediated drug reactions can result in anaphylaxis, Diff erential diagnosis and can be fatal. • Any new symptoms which occur aft er commencing a • Non-IgE- mediated reactions can also be extremely new drug should be considered as possibly drug-related. dangerous, and indeed fatal, e.g. Stevens–Johnson syn- • Given the array of diff erent cutaneous manifestations that drome and toxic epidermal necrolysis. may represent drug allergy or adverse drug reactions, skin Epidemiology biopsy is of limited value. However, biopsy may be useful to confi rm vasculitis or exclude other . • Allergic drug reactions are clearly under- reported, mak- • Peripheral eosinophilia, eosinophiluria, and acutely abnor- ing estimation of the incidence of drug allergy very diffi - mal liver function tests are suggestive of drug allergic cult. Anaphylaxis is, however, rare. reactions. • Patient self-reporting of drug allergy is unreliable. This • Skin-prick and intradermal allergy testing is a validated is particularly the case for antibiotics. Many patients (and tool for suspected allergy to penicillin and to local and practitioners) mistake the viral exanthem that is exacer- general anesthetic agents. Cephalosporin testing is also bated by the administration of antibiotics in childhood increasingly being performed. Positive tests have a for a drug allergy. high positive predictive value for IgE-mediated drug • Antibiotics remain the most common cause of drug allergy. allergy, especially those of the beta- lactam class. • Negative testing for penicillin should be followed up Pathophysiology with a carefully supervised oral challenge with one of the penicillin drugs. It is very unusual in this instance Drugs can cause hypersensitivity via type I, II, III and IV for anaphylaxis to occur, although delayed-onset rash reactions, although there is likely to be overlap between may occur, confi rming a non- IgE- mediated immuno- mechanisms in some instances. logical reaction. • Type I reactions are characterized by rapidity of onset, due to the presence of specifi c IgE to that drug on the surface of mast cells and basophils. CLINICAL PEARL • Type III (serum- sickness) reactions will oft en occur Cross-reactivity between IgE-mediated penicillin and 10–14 days aft er commencement of a drug. These reac- cephalosporin allergy is relatively uncommon, with esti- tions may last for up to 3 weeks, due to persistence of mated rates <5%. If anaphylaxis has occurred to one of immune complexes. the agents, however, it is prudent to seek specialty con- sultation and allergy skin testing prior to prescription of • Most drug allergic reactions are likely to be T-cell medi- the other class. Reactions less severe than anaphylaxis ated, and therefore fall within the type IV hypersensi- do not require testing, but observation of the patient tivity category. This explains the likelihood of onset of upon administration of the fi rst dose is advisable. symptoms aft er approximately 48 hours of drug use. • Type II reactions are less common. An example is Treatment hemolysis resulting from treatment with penicillin. In this instance, penicillin binds to erythrocyte membrane • Suspected or proven drug allergy necessitates strict proteins, resulting in antibody- directed attack against avoidance of the off ending drug. the cell and clearance by macrophages. • Patients should be advised to wear emergency identi- fi cation jewelry if anaphylaxis or other life-threatening Clinical features drug reactions have occurred. The clinical history, with very specifi c noting of all drugs • Emergency rapid desensitization is possible to some anti- taken and the timing of onset of symptoms and/or signs, biotics, e.g. penicillin, platinum and taxane chemother- is the key to making an accurate diagnosis of drug allergy. apy agents, aspirin and some biologic agents, if there is no Supplementary testing is unavailable, or inadequate, for all possible alternative for an inpatient with a life- threatening but a few drugs. problem. This procedure is high risk, and should only

524 Chapter 16 Immunology

Table 16-8 Specifi c serious drug reaction syndromes

COMMON SYNDROME CLINICAL FEATURES CULPRIT DRUGS TREATMENT

Stevens–Johnson Maculopapular rash, which may Allopurinol Medical emergency syndrome blister, ulcerate and necrose Carbamazepine Cease suspected culprit Mucosal ulceration (Figure 16-6) medications Fever Antibiotics, esp. Hemodynamic support Usually a drug reaction, but may sulfonamides Burns-style dressings and follow infection or malignancy, or be nursing care idiopathic Corticosteroids unproven but generally used in moderate to high doses

Toxic epidermal Fever Allopurinol Medical emergency necrolysis (TEN) Maculopapular rash Carbamazepine Cease suspected culprit Skin ulceration and necrosis aff ecting Phenytoin medications >30% of body surface area Antibiotics, esp. Hemodynamic support Mucosal ulceration sulfonamides Burns-style dressings and Usually a drug reaction, but may nursing care follow infection or malignancy, or be Corticosteroids unproven but idiopathic generally used in moderate to high doses, in conjunction with high-dose IV immunoglobulin

DRESS Fever Withdraw off ending drug (drug reaction with Onset >10 days after commencement Allopurinol Oral corticosteroids eosinophilia and of drug NSAIDs systemic symptoms) Maculopapular or vasculitic rash Sulfonamides Eosinophilia Abacavir Abnormal LFTs Interstitial nephritis

Erythema Urticaria Beta-lactam Cease suspected off ending drug multiforme Target lesions antibiotics Antihistamines if itch prominent Bullae Allopurinol Mucosal lesions may occur Carbamazepine Usually associated with infection, Phenytoin esp. herpesviruses, but may be a drug reaction antibiotics

IV, intravenous; LFT, liver function test; NSAID, non-steroidal anti-infl ammatory drug.

be performed by allergy specialists. It is also important to note that once the drug is discontinued or doses are missed, the patient’s sensitivity to the medication returns over days to weeks. Insect venom allergy Anaphylaxis to stinging insects is a life-threatening and unpredictable form of allergic disease. • Insects of the Hymenoptera order are the usual culprits. These are , yellowjackets/wasps, hornets and ants. Systemic allergic sting reactions have been reported in Figure 16-6 Oral mucosal involvement in Stevens– up to 7.5% of adults and 3.5% of children. Johnson syndrome • Anaphylaxis also can result from bites from a number of From Eichenfi eld LF et al. (eds). Neonatal and infant dermatology. species of fl y. 3rd ed. Philadelphia: Elsevier, 2015.

525 Essentials of internal medicine

• Neither nor family history is a risk factor for Eosinophils have multiple functions in innate immunity, Hymenoptera anaphylaxis, but repeated stings, e.g. in and infl ammation: beekeepers, does confer increased risk. • their production is stimulated by 5 and 3, and GM-CSF (-monocyte colony stimulat- Clinical features ing factor) Clinical reactions to Hymenoptera stings take three general • they release a variety of cytotoxic, chemotactic and forms: vasoactive molecules 1 Typical local reactions are localized pain, erythema, and • they are involved in tissue repair. edema. Pruritus may be prominent. Causes of eosinophilia are listed in Box 16- 9. 2 Large local reactions consist of extensive swelling and erythema, contiguous with the sting site. Even if an entire limb swells, it is classifi ed as a local reaction if Box 16-9 contiguous with the sting site. 3 Systemic reactions—can range from generalized urti- Causes of eosinophilia caria through to full- blown anaphylaxis. Common causes of eosinophilia Diff erential diagnosis • Allergic disease • Helminthic parasitic infection • It is usual for a patient to clearly have experienced the sting, due to pain, and a stinger will oft en remain in situ • Drug reactions following a bee or wasp sting. Primary eosinophilic disorders • The patient may have trouble distinguishing the insect, • Eosinophilic and may not recognize the species of wasp particularly. • Organ-specifi c eosinophilic syndromes, • Insect sting should be considered in any patient with e.g. ‘idiopathic’ anaphylaxis. • Eosinophilic fasciitis • Skin- prick and intradermal testing to diff erent Hyme- Disorders with secondary eosinophilia noptera species is very sensitive to diagnose sensitization. • Allergic bronchopulmonary aspergillosis Treatment • Eosinophilic granulomatosis with polyangiitis (Churg– Strauss vasculitis) The clinical history determines the appropriate course of treatment. • • Systemic mastocytosis • H1 antihistamines, NSAIDs and oral corticosteroids can be used for large local reactions. • Malignancy, e.g. lymphoma, • All patients with systemic reactions should have an • Hyper-IgE syndrome anaphylaxis action plan and carry an epinephrine self- • injection device. Eosinophilic clonal disorders • Myeloproliferative hypereosinophilic syndrome CLINICAL PEARL (M-HES) • Lymphoproliferative hypereosinophilic syndrome (L-HES) Subcutaneous immunotherapy to the specifi c insect venom is essential for any adult or child patient following • Eosinophilic leukemia (extremely rare) a systemic-allergic reaction exceeding generalized skin • Otherwise unclassifi ed HES symptoms with documented sensitization to the culprit insect venom with either specifi c serum IgE tests and/ or skin-prick tests. Immunotherapy reduces the risk of anaphylaxis with subsequent stings from >50% to <5%, Hypereosinophilic syndrome (HES) so is highly eff ective. Immunotherapy is neither eff ective • HES is defi ned as hypereosinophilia (>1.5 × 109/L eosin- nor appropriate for local reactions, however large. ophils in peripheral blood) or excessive tissue eosinophilia in the setting of clinical manifestations attributable to the eosinophilia, in the absence of an identifi able secondary EOSINOPHILIA cause. Eosinophilia is classifi ed according to the absolute level of • Patients present with a range of clinical problems includ- eosinophils in the peripheral blood: ing thromboembolism, pulmonary infi ltrates, endo- myocardial fi brosis, , , • mild eosinophilia: 0.5–1.5 × 109/L and myelofi brosis. • moderate eosinophilia: 1.5–5.0 × 109/L • Specifi c cytogenetic abnormalities are detectable in × 9 • severe eosinophilia: >5.0 10 /L. bone marrow in some subsets, including myeloprolifer- Hypereosinophilia is defi ned as a peripheral blood eosino- ative HES (M-HES) in which a tyrosine kinase muta- phil count of >1.5 × 109/L. tion is present.

526 Chapter 16 Immunology

• Corticosteroids are the mainstay of treatment for most Systemic mastocytosis (SM) variants of HES. • Skin involvement is seen in >50% of cases of SM. • M- HES is responsive to tyrosine kinase inhibitors, e.g. imatinib. • Systemic features typically manifest with pruritus, urti- • Anti- IL5 monoclonal antibody therapy is increasingly caria, fl ushing, diarrhea, abdominal pain, broncho- being used for all variants of HES and remains under spasm, and recurrent anaphylaxis. active clinical investigation. • Organ involvement can result in weight loss, lymph- adenopathy, and hepatosplenomegaly. Bone mar- row involvement is most common outside of skin MAST CELL DISORDERS involvement. Cutaneous and systemic mastocytosis represent a spectrum • Cardiac and neurological involvement is rare, but has of malignant mast cell disorders. They are predominantly been described. due to mutations in the KIT-oncogene, which results in • Bone involvement with either osteopenia or osteo- abnormal activation of the gene with uncontrolled prolifer- sclerosis is frequent. ation of mast cells. • Serum tryptase is elevated. Cutaneous mastocytosis (CM) • Cytopenias may result from bone marrow infi ltration. • Diagnosis is confi rmed with excess mast cells seen on • Cutaneous mastocytosis is the most common presen- bone marrow biopsy. The majority of patients will have tation of mast cell disease, representing 90% of cases. somatic mutations in the KIT gene (CD117 receptor) as Urticaria pigmentosa produces a raised red, brown or well as CD25 expression on clonal mast cells. purple papular rash (Figure 16-7) which can occur any- where other than the face. The rash urticates on scratch- ing (Darier’s sign). Treatment

• It is due to the accumulation of mast cells in the skin. • High- dose H1 and H2 antihistamine therapy is the • Approximately 25% of those with CM will have sys- mainstay of treatment. temic symptoms, but without demonstrable mast cell • UV therapy may off er some relief for urticaria pigmentosa. accumulation in tissues other than the skin, most nota- • Oral sodium cromolyn should be used for gastrointestinal bly the bone marrow. symptoms, and may relieve some systemic symptoms by • CM aff ects both children and adults. The childhood preventing mast cell degranulation. Leukotriene antago- form may regress spontaneously. nists may be useful as second line for pruritus. • Diagnosis is made by skin biopsy. • The oral multiple tyrosine kinase inhibitor has been shown to be highly eff ective in patients with advanced SM. Other tyrosine kinase inhibitors such as imatinib are only eff ective in patients without a KIT and thus are not appropriate for a majority of patients. • Corticosteroids may relieve symptoms in some cases of SM. • Aggressive cases of SM are usually treated with cyto- toxic chemotherapy. • Anaphylactic reactions are common, occurring in approximately half of all adult SM patients. Patients with SM should carry an epinephrine self- injecting device if there is a history of anaphylaxis. • All patients should be aware of potential triggers of mast cell degranulation in particular Hymenoptera stings, infection, trauma, emotional stress and drugs such as alcohol, aspirin, NSAIDs, opioids and general anes- thetic agents. • SM patients can suff er severe anaphylaxis in response to insect stings. All patients with anaphylaxis to Hymenop- Figure 16-7 Typical appearance of pigmented rash in tera sting should be off ered venom immunotherapy if urticaria pigmentosa, and demonstrating Darier’s sign found to be sensitized. From Skarin AT (ed). Atlas of diagnostic , 4th ed. • Pre-medication with antihistamines and corticosteroids Philadelphia: Elsevier, 2010. is usually administered prior to surgical procedures.

527 Essentials of internal medicine

• Omalizumab should be considered in patients with • direct antibody- directed cellular cytotoxicity severe symptoms such as recurrent anaphylaxis unre- • formation with resultant tissue . sponsive to the aforementioned treatment measures. Immunological abnormalities which have been demon- • Allogeneic hematopoeitic stem cell transplant is the strated in SLE include: only potential for in patients with advanced SM. • abnormalities in T cell number and function • SM patients should wear emergency identifi cation jewelry. • an excess of autoreactive B cells but with overall B cell lymphopenia, and hyperactivation of the BAFF path- way, a T cell independent B cell survival pathway SYSTEMIC AUTOIMMUNE • reduced number and function of natural killer cells • high levels of circulating immune complexes, and DISEASE decreased clearance by the reticuloendothelial system • The diagnosis of one of the forms of systemic autoim- • abnormal cytokine profi les, including strong type 1 mune disease is frequently devastating for a patient. This interferon signature group of diseases is chronic and incurable. Treatment is • excess and impaired degradation of extracel- oft en associated with signifi cant short- and long- term lular traps, resulting in increased nucleic acid antigen morbidity. production. • Diagnostic uncertainty is an issue in many cases, with Known risk factors aside from gender include: patients potentially displaying clinical features which overlap between conditions, and laboratory investi- • Inherited complement component defi ciencies (C1q, gations with imperfect sensitivity and specifi city (see C2, C4). Complement defi ciency results in reduced Table 16- 9 later in the chapter). ability to clear apoptotic cells, with consequent increased exposure of cellular material on cell- surface • The relatively low prevalence of most systemic auto- blebs to immunologically active cells, and the formation immune diseases, combined with the heterogeneity of of autoantibodies. clinical patterns, makes development of precise treat- ment protocols diffi cult. Clinicians will need to individ- • Family history. Monozygotic twins have an approxi- ualize their approach in all cases. mate 1 in 4 chance of disease concordance, with only 3% of dizygotic twins aff ected. The following systemic autoimmune conditions will be considered in this section: • Certain HLA alleles. HLA sub types A1, B8, DR2, and DR3 show an increased frequency in SLE patients. • systemic lupus erythematosus • Sjögren’s syndrome • Certain drugs. Drug-induced lupus is associated with multiple medications, including , carba- • mazepine and isoniazid, and newer biological agents • dermatomyositis such as infl iximab and etanercept. • scleroderma • Race. SLE is more common in Southeast Asians and • CREST syndrome African- Americans. • mixed connective tissue disease Flares in disease may be associated with situations in which • primary antiphospholipid syndrome there is higher than normal cell damage or turnover, such • IgG4- related disease. as aft er exposure to UV radiation, or infection, or during pregnancy. The disease may manifest with exacerbations is covered in Chapter 17. and remissions, or pursue a relentless course.

Systemic lupus erythematosus (SLE) Diagnosis Systemic lupus erythematosus is a multisystem disorder of Diagnosis is based on: unknown etiology, thought to aff ect approximately 1 in 1000 people. Females outnumber males in a ratio of 8–10:1. • the totality of clinical manifestations Rare in childhood but not unheard of, onset is frequently • autoimmune serology in the young adult years. Manifestations of the disease are • tissue diagnosis where possible and appropriate; histo- protean, and can be mild through to lethal (Box 16- 10). is particularly important in the setting of renal disease (see Chapter 9), and with skin disease where the Pathophysiology diagnosis is not clear. Systemic lupus erythematosus results from the formation Criteria have been developed in an attempt to increase the of pathogenic autoantibodies targeting nucleic acid-bound precision of diagnosis, and are important when attempting antigens. Much of the tissue damage is thought to result to standardize patient populations for research and treatment from: protocols. Those given in Box 16- 11 are from the Systemic • the formation of immune complexes, subsequent Lupus International Collaborating Clinics (2012), which deposition, and initiation of infl am- has high sensitivity (94.6%) and specifi city (95.5%) for the mation via complement and Fc- receptor activation diagnosis of adult SLE.

528 Chapter 16 Immunology

Box 16-10 Clinical features of systemic lupus erythematosus (SLE)

Incidence of clinical manifestations in SLE (in • Hematological—normocytic anemia, hemolysis, approximate descending order) lymphopenia, thrombocytopenia • (or arthritis) • Serositis—pericarditis, pleuritis • Fatigue • Raynaud’s phenomenon • Cutaneous—malar rash (Figure 16-8), photosensitivity, • Oral ulceration discoid lupus (Figure 16-9), alopecia, cutaneous • Neuropsychiatric—cerebritis, cerebral vasculitis, vasculitis, subacute sclerosing lesions , psychosis, seizures • Myalgia (or uncommonly myositis) • Gastrointestinal—hepatitis, intestinal vasculitis • Renal—recurrent urinary tract infection, • Anti-phospholipid-related—recurrent miscarriage, glomerulonephritis • Pulmonary—pneumonitis, pleuritis • Cardiac—pericarditis, non-infective • Generalized lymphadenopathy

Figure 16-8 This picture displays the malar (or Figure 16-9 A discoid lupus lesion on the face. Note butterfl y) rash typical of SLE. Note the sparing of the predilection for the face, scaly appearance, and the nasolabial folds, which distinguishes it from lighter color in the center of the lesion, as is typical seborrheic dermatitis in discoid lupus From Wallace D and Hahn BH (eds). Dubois’ Lupus erythematosus From Tsokos C, Gordon C and Smolen JS (eds). Systemic lupus and related syndromes, 8th ed. Philadelphia: Elsevier, 2013. erythematosus. St Louis: Elsevier, 2007.

Autoantibodies in SLE Demonstration of the presence of autoantibodies helps sig- CLINICAL PEARL nifi cantly with diagnosis and classifi cation of systemic auto- Patients with nonspecifi c symptoms such as fatigue immune disease. However, clinical features remain the ‘gold and arthralgia are often labeled as having ‘a mild case of standard’ for diagnosis, as: lupus’. They may be an adult female with a detectable • a signifi cant minority of the population will have detect- low-titer antinuclear antibody in the absence of any objective clinical signs or specifi c laboratory abnor- able autoantibodies in the absence of any autoimmune malities. Such spurious diagnostic labeling can have a disease profoundly negative long-term eff ect on an individual, • up to 20% of asymptomatic relatives of SLE patients will with resultant unnecessary investigation and excessive have detectable antinuclear antibodies (ANA) specialist consultation. Potentially, iatrogenic harm will • the presence of low-titer ANA increases with age, espe- result from inappropriate use of toxic . Always consider non-immunological physical or psychologi- cially in women. cal causes when faced with this type of scenario. Conversely, SLE is very rare in the absence of ANA, so caution must be exercised when diagnosing ‘ANA- negative lupus’.

529 Essentials of internal medicine

Box 16-11 SLICC classifi cation criteria for systemic lupus erythematosus (SLE)

Requirements: ≥4 criteria (at least 1 clinical and 1 laboratory criterion) or biopsy-proven lupus nephritis with positive ANA or anti-DNA. *See notes for criteria details.

Clinical criteria Immunological criteria 1 Acute cutaneous lupus* 1 ANA 2 Chronic cutaneous lupus* 2 Anti-DNA 3 Oral or nasal ulcers* 3 Anti-Smith antibodies 4 Non-scarring alopecia 4 Antiphospholid antibodies 5 Arthritis* 5 Low complement (C3, C4, CH50) 6 Serositis* 6 Direct Coombs’ test (do not count in the presence of 7 Renal* hemolytic anemia) 8 Neurological* 9 Hemolytic anemia 10 * 11 Thrombocytopenia (<100,000/mm3)

Notes CLINICAL CRITERIA 6. Serositis 1. Acute cutaneous lupus OR subacute cutaneous lupus • Typical pleurisy for more than 1 day OR pleural eff usions OR • Acute cutaneous lupus: lupus malar rash (do not count if malar pleural rub discoid), bullous lupus, toxic epidermal necrolysis variant of • Typical pericardial pain (pain with recumbency improved by SLE, maculopapular lupus rash, photosensitive lupus rash (in sitting forward) for more than 1 day OR pericardial eff usion OR the absence of dermatomyositis) pericardial rub OR pericarditis by electrocardiography • Subacute cutaneous lupus: nonindurated psoriaform and/ • In the absence of other causes, such as infection, uremia and or annular polycyclic lesions that resolve without scarring, Dressler’s pericarditis although occasionally with postinfl ammatory dyspigmentation 7. Renal or telangiectasias) • Urine protein-to-creatinine ratio (or 24-hour urine protein) 2. Chronic cutaneous lupus representing 500 mg protein/24 hours OR red casts • Classic discoid rash localized (above the neck) or generalized 8. Neurological (above and below the neck), hypertrophic (verrucous) lupus, lupus (profundus), mucosal lupus, lupus • Seizures, psychosis, mononeuritis multiplex (in the absence erythematosus tumidus, chillblains lupus, discoid lupus/lichen of other known causes such as primary vasculitis), , planus overlap peripheral or cranial neuropathy (in the absence of other known causes such as primary vasculitis, infection and diabetes 3. Oral ulcers OR nasal ulcers mellitus), acute confusional state (in the absence of other • Oral: , buccal, tongue causes, including toxic/metabolic, uremia, drugs) • Nasal ulcers 9. Hemolytic anemia • In the absence of other causes, such as vasculitis, Behçet’s 10. Leukopenia (<4000/mm3) OR lymphopenia (<1000/mm3) disease, infection (herpesvirus), infl ammatory bowel disease, • Leukopenia at least once in the absence of other known causes , and acidic foods such as Felty’s syndrome, drugs, and 4. Non-scarring alopecia • Lymphopenia at least once in the absence of other known • Diff use thinning or hair fragility with visible broken hairs, in the causes such as corticosteroids, drugs, and infection absence of other causes such as , drugs, iron 11. Thrombocytopenia (<100,000/mm3) defi ciency, and androgenic alopecia • At least once in the absence of other known causes such as 5. Synovitis involving 2 or more joints drugs, portal hypertension, and thrombotic thrombocytopenic • Characterized by swelling or eff usion purpura • OR tenderness in 2 or more joints and at least 30 minutes of morning stiff ness

530 Chapter 16 Immunology

IMMUNOLOGICAL CRITERIA • False-positive test result for rapid plasma reagin 1. ANA level above laboratory reference range • Medium- or high-titer anti-cardiolipin antibody level (IgA, IgG or 2. Anti-dsDNA antibody level above laboratory reference range (or IgM) 2-fold the reference range if tested by ELISA) • Positive test result for anti-beta-2 glycoprotein 1 (IgA, IgG or 3. Anti-Sm: presence of antibody to Sm nuclear antigen IgM) 4. Antiphospholipid antibody positivity, as determined by 5. Low complement (C3, C4 or CH50) • Positive test for lupus anticoagulant 6. Direct Coombs’ test (in the absence of hemolytic anemia)

ANA, antinuclear antibodies; anti-dsDNA, anti-double stranded DNA; anti-Sm, anti-Smith antibody; DNA, deoxyribonucleic acid; dsDNA, double-stranded DNA; Ig, immunoglobulin; SLICC, Systemic Lupus International Collaborating Clinics Collaboration. From Petri M et al. Derivation and validation of Systemic Lupus International Collaborating Clinics classifi cation criteria for systemic lupus erythematosus. Arthritis Rheum 2012;64(8):2677–86.

Table 16-9 Autoantibodies found in systemic lupus erythematosus (SLE)

AUTOANTIBODY SENSITIVITY COMMENT Antinuclear antibodies (ANA) >95% More specifi c with increasing titer Titers <1:320 not considered signifi cant in most laboratories Homogeneous pattern more specifi c for SLE No relationship with disease activity Double-stranded DNA (dsDNA) ~50% Good specifi city for SLE, which increases with higher titer Titer may refl ect disease activity Risk factor for renal disease SS-A (Ro) 30–50% Also found in Sjögren’s syndrome Associated with subacute cutaneous disease 2% of babies born to SS-A-positive mothers develop congenital heart block SS-B (La) 10% More sensitive and specifi c for SS Anti-Sm (Smith) 20–30% Highly specifi c for SLE Anti-histone High for drug-induced Not found in association with anti-TNF-therapy-associated lupus drug-induced lupus (infl iximab, etanercept) Anti-cardiolipin antibodies 25% May be associated with anti-phospholipid syndrome, or (including lupus inhibitor) asymptomatic TNF, .

Table 16- 9 describes autoantibodies found in SLE. • All but the cardiac lesions resolve by 3 months of age (commensurate with the half- life of maternal IgG). Neonatal lupus Monitoring of disease activity • Neonatal lupus results from the passage of IgG anti-SSA (Ro) antibodies from mothers with SLE or Sjögren’s Clinical history and physical examination will provide syndrome to the fetus via the placenta. the most accurate gauge of disease activity, but laboratory investigations are a useful supplement. A patient with active • Clinical features in the newborn include complete heart infl ammatory disease will characteristically demonstrate evi- block, which is frequently irreversible and may be asso- dence of an acute-phase response and active consumption of ciated with congestive if the heart block complement. develops suffi ciently early before delivery; skin rash; and A disease fl are may be accompanied by: hematological abnormalities including hemolytic ane- mia and thrombocytopenia. • a rising titer of anti- ds- DNA antibodies • an elevated erythrocyte sedimentation rate (ESR) • Severity of maternal illness does not correlate with the risk of development of neonatal lupus, but the risk is • normochromic, normocytic anemia signifi cant if previous were aff ected. • active urinary sediment

531 Essentials of internal medicine

• reduced levels of complement components C3 and C4, • inhibitors including tacrolimus and and total hemolytic complement. cyclosporine have been shown to be eff ective for lupus nephritis. CLINICAL PEARL Several biological agents show great promise in the treat- ment of lupus. C-reactive protein (CRP) is often not elevated in the • Belimumab is a monoclonal antibody against BAFF (B setting of systemic lupus erythematosus. A rise in CRP in a lupus patient with a previously normal CRP should cell activating factor of the TNF family), an important raise the prospect of concurrent infection. T cell independent B cell survival factor. It is eff ective in the treatment of non-renal lupus and also signifi cantly reduces fl ares. Treatment and prognosis • Rituximab is an anti-CD20 monoclonal antibody that In general terms, treatment for SLE is symptomatic for has shown clinical effi cacy in lupus nephritis patients manifestations that are non- organ- or non- life- threatening, refractory to conventional therapy; however, data but immunosuppressive for the more serious complications. remains controversial. • Non- steroidal anti- infl ammatory drugs (NSAIDs) may All patients should be advised to avoid known disease trig- be helpful for arthralgia and myalgia, and sometimes for gers including UV, through adequate sun protection and pericarditis. use of at least SPF50 sunscreen. Infections are common and • has important immune modulation patients should receive infl uenza and pneumococcal vaccina- and disease-modifying actions and is the only drug shown tions in addition to the local immunization schedule. Oste- to improve survival in SLE patients, as well as reduce lupus oporosis is more frequent in SLE patients, and bone health fl ares and prevent organ damage. It should be used for should be optimized in all patients. Table 16-10 summarizes every SLE patient unless there is a clear contraindication, the mechanism of action and major toxicities of drugs used at a dose of 200–400 mg daily. Hydroxychloroquine also to treat SLE. Given the chronic nature of SLE, much of the improves cutaneous manifestations and arthritis. long-term morbidity is due to the eff ects of treatment, par- ticularly corticosteroids. Table 16-11 provides a guide that • Vitamin D should be supplemented in all patients with may be useful in systematic monitoring of patients on long- insuffi ciency, and has anti-fi brotic and immunomod- term steroids and/or immunosuppressants. ulatory eff ects associated with improved global disease Patients with SLE have a 5- year survival rate of >90%. activity. Given the heterogeneity of the condition, prognostication • Corticosteroids have been widely used for their anti- is diffi cult, and clearly is worse in those with signifi cant infl ammatory and immunosuppressive actions, with renal, neurological or pulmonary disease. The major long- higher doses used as initial, urgent therapy particularly term risk to health is likely to be due to medication toxicity, for nephritis, neuropsychiatric disease or severe cytope- through increased risk of: nias. However, emerging studies suggest corticosteroids • malignancy might not be necessary to control lupus manifestations and there is an increasing move to avoid oral corticoster- • infection oids as much as possible due to the signifi cantly increased • metabolic disease. risk of cardiovascular events and later organ damage. There is some evidence that SLE is an independent risk fac- • Intravenous (IV) high-dose methylprednisolone is the tor for atherosclerotic vascular disease, presumably due to most rapidly-acting immunosuppressive agent available, the atherogenic consequences of chronic infl ammation. The but does carry the risk of causing avascular osteonecro- risk of cardiovascular events is increased by 2.66-fold and sis, particularly of the femoral head. aggressive management of modifi able cardiovascular risk • is a highly toxic alkylating agent factors is essential. that was widely used in the past for the induction and Sjögren’s syndrome (SS) maintenance treatment of severe forms of lupus such as lupus nephritis and neuropsychiatric disease. It has, Primary Sjögren’s syndrome is a chronic, relentless, systemic however, now been largely replaced by less toxic immu- autoimmune disease characterized by autoimmune destruc- nosuppressive medications. tion of exocrine glands, and sometimes accompanied by vas- culitis. The classic clinical features of SS are: There are now several less toxic, eff ective immunosuppres- sive agents, as well as biological agents. • dry eyes due to keratoconjunctivitis sicca, secondary to destruction of the lacrimal glands • Mycophenolate is eff ective for the induction and main- tenance of lupus nephritis, with similar effi cacy com- • dry mouth (xerostomia), due to destruction of the pared to cyclophosphamide. It has also been shown to salivary glands. be superior to azathioprine for controlling disease activ- The disease is generally not life- threatening, but serious ity and preventing both renal and non-renal fl ares. complications such as nephritis and pneumonitis can occur • Azathioprine has been used for the treatment of renal (Table 16- 12). and extra-renal SLE for many decades, and remains an • It should be kept in mind that patients have a signifi - excellent option to control disease activity in pregnancy. cantly increased risk of lymphoma.

532 Chapter 16 Immunology

Table 16-10 Mechanism of action and major toxicities of drugs commonly used to treat SLE

DRUG MECHANISM OF ACTION MAJOR TOXICITIES Hydroxychloroquine Inhibits B cell receptor and toll-like receptor Retinal toxicity signaling, interferes with MHC-antigen binding by increasing lysosomal pH, which aff ects autoantigen processing and cytokine secretion Has anti-type 1 interferon activity Prednisolone Inhibits leukocyte migration Weight gain, osteoporosis, glucose Reduces cytokine production intolerance/diabetes mellitus, hypertension, Cushing’s syndrome, poor wound Suppresses antibody production healing, avascular osteonecrosis, immunosuppression Azathioprine Inhibits DNA synthesis in dividing cells, therefore is Hepatitis, GI intolerance, myelosuppression, cytotoxic to lymphocytes skin malignancy, immunosuppression Blocks CD28 co-stimulation, so prevents activation of naïve lymphocytes Methotrexate Inhibits enzyme dihydrofolate reductase, which GI tract disturbance, mucositis, leads to impairment of thymidylate synthesis, and myelosuppression, hepatic fi brosis, impairment of DNA synthesis pulmonary fi brosis, immunosuppression The anti-infl ammatory eff ect results from the impairment of leukocyte function Cyclosporine Inhibits production of IL-2 by T lymphocytes, thus Renal impairment, hypertension, glucose (ciclosporin) reducing induction of cytotoxic T cells and T-cell- intolerance, gum hypertrophy, hirsutism, dependent B cell responses immunosuppression, skin malignancy Cyclophosphamide Alkylating agent which cross-links DNA Myelosuppression, especially neutropenia, Has a pronounced eff ect on reducing the function hemorrhagic cystitis, nausea and of lymphocytes vomiting, bladder cancer, skin malignancy, immunosuppression, pulmonary fi brosis Mycophenolate Inhibits purine synthesis, resulting in reduced GI tract disturbance, anemia, guanine nucleotides especially in lymphocytes, immunosuppression resulting in reduced function

GI, gastrointestinal; IL, interleukin; SLE, systemic lupus erythematosus.

• When xerophthalmia or xerostomia occurs in associa- Diagnosis tion with other systemic autoimmune disease, such as Diagnosis is generally based on the American-European rheumatoid arthritis or SLE, this is known as secondary consensus group (AECG) classifi cation, with SS diagnosed Sjögren’s syndrome, which is generally less severe than pri- when 4 of the following 6 items are present: mary Sjogren’s syndrome. 1. Subjective presence of ocular dryness Pathophysiology 2. Subjective presence of oral dryness 3. Objective measures of ocular dryness by Schirmer’s test • The etiology of SS is unknown. or corneal staining • It is predominantly a disease of middle- aged women. 4. Focus score >2 in salivary gland biopsy • It is likely that by the time of diagnosis, most of the dam- 5. Salivary scintigraphy showing reduced salivary fl ow and/ age to the exocrine glands has been done. This explains or diff use the lack of effi cacy of immunosuppressive medication 6. Positive autoantibodies against SS-A and/or SS-B. in this condition, which is generally contraindicated Either salivary gland biopsy or presence of autoantibodies is except when vasculitis occurs. mandatory. • Histopathology reveals intense lymphocytic infi ltration The classic autoimmune serology found in SS consists of: of involved structures in SS. • high- titer antinuclear antibody (ANA) in speckled • The tissue damage is likely a result of type IV (cell- pattern. mediated) immune hypersensitivity. • positive SS- A and SS- B (Ro and La) antibodies • The autoantigen is unknown. • positive .

533 Essentials of internal medicine

Table 16-11 Monitoring of patients on long-term immunosuppressants

DRUG EVERY 3 MONTHS EVERY 6 MONTHS ANNUALLY Corticosteroids Blood sugar levels Lipid monitoring Bone densitometry Blood pressure monitoring Cyclophosphamide Full blood count (more frequently Urinalysis, for Skin examination (for pre-malignant depending on protocol being hematuria; cystoscopy and malignant change) used) if positive Papanicolaou (Pap) tests (females) Azathioprine Liver function tests Skin examination (for pre-malignant Full blood count and malignant change) Pap tests (females) Methotrexate Liver function tests Skin examination (for pre-malignant Full blood count (more frequently and malignant change) in initiation of treatment phase) Pap tests (females) Clinical screening for cough or dyspnea (risk of pulmonary fi brosis) Cyclosporine Blood sugar levels Lipid monitoring Skin examination (for pre-malignant (ciclosporin) Blood pressure monitoring and malignant change) Renal function and electrolytes Pap tests (females) Oral examination (for gum Biannual renal DTPA scans hypertrophy) Mycophenolate Full blood count Skin examination (for pre-malignant and malignant change) Pap tests (females)

DTPA, diethylene triamine pentaacetic acid.

Table 16-12 Major clinical manifestations of Sjögren’s syndrome

MANIFESTATION CAUSE Keratoconjunctivitis sicca Destruction of lacrimal glands Xerostomia Destruction of salivary glands Dry skin Destruction of sebaceous glands Dry cough Due to lack of respiratory tract mucus Dental caries//periodontitis Due to lack of saliva May have recurrent painful episodes, or chronic infl ammation with gradual parotidomegaly Be aware of possibility of malignant transformation Rare Pneumonitis May lead to ‘shrinking lung syndrome’ Cerebral vasculitis Likely to result in focal neurological signs, or organic brain syndrome Interstitial nephritis Renal tubular acidosis a likely outcome. Can progress to renal failure Glomerulonephritis May progress to end-stage renal disease Cutaneous vasculitis Often aff ects lower limbs, and tends to be recurrent Fatigue Lymphoma Most often in parotid glands

534 Chapter 16 Immunology

It is typical to fi nd: • elevated creatine kinase in the vast majority of cases • signifi cant polyclonal • characteristic electromyographic changes of fi brillations • markedly elevated erythrocyte sedimentation rate (ESR). and positive sharp waves. It is advisable for the physician to involve an ophthalmol- These diseases are rare and approximately twice as common ogist in the long- term care of the SS patient. in women as men. The etiology is unknown, although there are associations with certain HLA alleles. Onset is more Treatment common in middle age, but can occur in childhood. Treatment is largely symptomatic. Anti-synthetase syndrome • Preservative-free lubricant eyedrops such as methylcel- lulose are important both for comfort and to prevent • This is a separate subgroup of IIM that is associated with scarring of the eye surface. They should be applied the presence of anti-synthetase antibodies. every couple of hours during the day. • 75% of patients will have Jo-1 antibodies. • Lubricant ocular gel can be applied prior to bed. • Other antibodies include PL-7; PL-12; EJ; OJ; KS; Zo. • The syndrome includes interstitial lung disease, non- CLINICAL PEARL erosive arthritis, and a characteristic hyperkeratotic der- matosis known as mechanic’s hands (Figure 16-10). Cyclosporine (ciclosporin) eyedrops can be quite suc- • Raynaud’s phenomenon may be prominent, and fever cessful at restoring some tear production and reducing may occur. the severity of keratoconjunctivitis sicca, which is inter- esting given the lack of effi cacy of systemic immuno- suppression in SS. Diagnosis The diagnosis can be established to a satisfactory degree of • Dry mouth is diffi cult to treat. certainty with the combination of: » Patients may benefi t from using an atomizer to spray • clinical features water into the mouth if frequent water-sipping • elevated creatine kinase levels causes polyuria. • characteristic electromyographic features » Pilocarpine mouth washes may help stimulate resid- ual salivary gland function. • positive autoimmune serology is helpful in defi ning the » Regular dental care is essential to attempt to avoid clinical phenotype progressive tooth damage. • rheumatoid factor positivity in approximately 50% of patients • Methotrexate and hydroxychloroquine are used for the • muscle biopsy remains the gold standard for diagnosis treatment of arthritis in pSS patients. and diff erentiation of the subgroups of IIM • There is some evidence that rituximab shows benefi cial • MRI has become increasing useful for diagnosis, guiding eff ects on B cell activity, glandular morphology, fatigue, muscle biopsies and monitoring response to treatment. dryness and several extraglandular manifestations, and may be considered for manifestations. Patients should be regularly monitored for development of complications such as interstitial nephritis or glomerulone- phritis, although unfortunately immunosuppression is fre- quently ineff ective at altering disease course. Lymphoma needs to be kept in mind as a long- term risk. Infl ammatory myopathies The classifi cation of idiopathic infl ammatory myopathies (IIM) is evolving and now includes the following subgroups: sporadic inclusion-body myositis; polymyositis; dermatomy- ositis; non-specifi c (or overlap) myositis, and immune-me- diated necrotizing myopathy. A comparison of the diff erent subgroups is summarized in Table 16-13. Patients typically present with: • progressive symmetrical, usually proximal, of skeletal muscle • muscle aching or pain, although this is variable Figure 16-10 The hyperkeratosis and fi ssuring known • cutaneous changes in the case of dermatomyositis as mechanic’s hands • clinical features and autoantibodies that overlap with From James WD, Berger T and Elston D. Andrews’ Diseases of the other connective tissue disorders in overlap myositis skin: clinical dermatology, 11th ed. Elsevier, 2011.

535 Essentials of internal medicine

Table 16-13 Infl ammatory myopathies

IMMUNE-MEDIATED INCLUSION BODY POLYMYOSITIS NECROTIZING MYOSITIS (IBM) (PM) DERMATOMYOSITIS (DM) MYOPATHY (IMNM)

No skin involvement No skin involvement Pathognomonic cutaneous signs: No skin involvement — Violaceous (heliotrope) rash on and around top eyelids (Figure 16-11) — Gottron’s nodules/papules over extensor surfaces of hands (Figure 16-12) Non-pathognomonic cutaneous signs: — Facial erythema — Poikiloderma (Figure 16-13)

May start asymmetrically. Progressive Progressive symmetrical muscle pain Signifi cant proximal Hip fl exors, quadriceps and symmetrical muscle and weakness symmetric weakness fi nger fl exors preferentially pain and weakness aff ected. More common in men

Muscle biopsy shows Muscle biopsy Muscle biopsy shows muscle fi ber Muscle biopsy shows internalization of nuclei shows perifascicular infi ltration by cytotoxic (CD8+) necrotic fi bers with in muscle fi bers, CD8+ muscle atrophy T cells, and macrophages macrophages. Absence of T cells; congophilic and regeneration; CD8+ T cells. amyloid deposits; scant infi ltration with infl ammation CD4+ T cells

Antibodies against Not currently Antibodies against Mi-2; SAE; MDA5; Antibodies against SRP; Mup44; Anti-cytosolic associated with any TIF1-gamma; TIF1-beta; NXP2 HMG-Co A reductase 5’-Nucleotidase 1A (cN1A) myositis-specifi c (not routinely assessed) antibodies

Malignancy is rarely Increased risk of Signifi cantly increased risk of Increased risk of associated malignancy, but less malignancy compared with general malignancy, but less so so than in DM population than in DM

Figure 16-11 Heliotrope rash around the eyes in a patient with dermatomyositis From Paller AS and Mancini AJ. Hurwitz Clinical paediatric dermatology, 4th ed. Philadelphia: Elsevier, 2011.

Treatment • Corticosteroids are the mainstay of treatment and are generally eff ective, but oft en at unacceptably high doses with excessive toxicity. • A steroid-sparing agent such as methotrexate, azathio- prine, mycophenolate or rituximab is frequently required. Figure 16-12 Gottron’s papules overlying the • High-dose IV immunoglobulin is a treatment for which metacarpophalangeal, proximal interphalangeal, and there is an evidence base. It does not result in immuno- distal interphalangeal joints suppression, so is useful if eff ective. From Paller AS and Mancini AJ. Hurwitz Clinical paediatric • IBM is typically resistant to immunosuppression. dermatology, 4th ed. Philadelphia: Elsevier, 2011.

536 Chapter 16 Immunology

Box 16-12 Skin changes in scleroderma • Thickening and edema of digits (sclerodactyly) (Figure 16-14) • Prominent telangiectasiae, especially on digits and face • Skin tightening on face, resulting in reduction in oral aperture, and ‘beaking’ of nose • Calcinotic nodules and/or ulcers, especially on digits • Pulp atrophy of digits • Thickening and edema of skin on trunk

Figure 16-13 Poikiloderma, as may be found in dermatomyositis. Note the combination of hyper- and hypo-pigmentation, and erythema From Bolognia JL et al. Dermatology, 2nd ed. St Louis: Elsevier, 2008.

CLINICAL PEARL

Steroid-sparing immunosuppressive agents will fre- Figure 16-14 The typical swollen and shiny quently be required in the setting of systemic auto- immune disease, given that these diseases are chronic appearance of hands due to sclerodactyly. Note and often life-threatening. The rarity, and heterogene- also the fi xed fl exion position of the fi ngers ity, of systemic autoimmune disease limits the availabil- From Marks JG and Miller JJ. Lookingbill & Marks’ Principles ity of evidence-based treatment guidelines for many of dermatology, 4th ed. Philadelphia: Elsevier, 2006. clinical scenarios, unlike in common diseases such as hypertension or ischemic heart disease. The skill in managing systemic autoimmune disease lies therefore in striking a balance between effi cacy, short- and long- term toxicity, patient adherence to treatment, drug Box 16-13 cost and availability, and individual patient factors. For example, a young female with dermatomyositis who Organ involvement in scleroderma has become Cushingoid with high doses of prednis- Pulmonary olone may be best started on azathioprine rather than cyclosporine (ciclosporin) or mycophenolate, because: • Interstitial fi brosis • there is concern that she may not be fully adherent • Pulmonary arterial hypertension with contraception (mycophenolate is highly terato- • Pleural eff usion genic, azathioprine is better established as safer in Gastrointestinal pregnancy than cyclosporine) • her blood pressure is 125/85 mmHg (cyclosporine is • Refl ux esophagitis likely to make this worse) • Dysphagia due to strictures • there is a strong family history of type 2 diabetes melli- • Intestinal fi brosis leading to dysmotility and absorptive tus (cyclosporine predisposes to glucose intolerance). abnormalities • Vascular ectasia in the ( stomach) Renal Scleroderma and CREST syndrome • Accelerated hypertension • Rapidly progressive renal failure due to renal Also known as diff use and limited systemic sclerosis, this is a vasculopathy disease spectrum involving autoimmune infl ammation with vascular involvement and connective tissue scarring in the Cardiac skin and multiple other organs. It is thought to have an inci- • Pericarditis dence of around 1 in 10,000. • • Cardiac failure Clinical manifestations • Diff use scleroderma nearly always involves the skin both • In limited scleroderma, skin involvement is usually con- below and above the elbows and knees, including the fi ned to the distal limbs, although facial changes can face and trunk (Box 16-12). Patients are at increased occur. The label CREST syndrome is oft en given, due to risk of pulmonary, gastrointestinal, renal and cardiac the prominence of Calcinosis, Raynaud’s phenomenon, involvement (Box 16- 13). Esophageal dysfunction, Sclerodactyly and Telangiectases.

537 Essentials of internal medicine

• CREST patients will oft en have digital ulcers, and are at risk of pulmonary arterial hypertension. Box 16-14 • Nail- fold capillaroscopy will reveal abnormalities in Diff erential diagnosis of thickened most patients with both limited and diff use disease. and tethered skin Changes include reduction in numbers of capillaries, with those remaining forming giant loops. • CREST syndrome, scleroderma, or mixed connective tissue disease • Raynaud’s phenomenon is oft en an early manifestation, occurs almost invariably, and refl ects the vascular abnor- • Eosinophilic fasciitis mality with propensity to vasospasm. This contributes • Localized morphea—small areas of sclerosis to digital ulceration in around half of cases, and can be • Chemicals—vinyl chloride, pentazocine, bleomycin, further complicated by digital , infection with toxic oil syndrome osteomyelitis, and autoamputation. • Pseudoscleroderma—secondary to porphyria cutanea • Digital ulceration is extremely painful, and disabling. tarda, acromegaly, carcinoid syndrome The etiology of scleroderma is not known and the patho- • Scleredema—diabetics develop thick skin over the shoulders and upper back genesis is complex, with abnormalities demonstrable in the function of endothelium, epithelium, fi broblasts and lym- • Graft-versus-host disease phocytes. The end result is accumulation of abnormal and • Silicosis excessive extracellular matrix, which gradually fi broses and causes tissue dysfunction. A variety of autoantibodies are • Autologous human stem- cell transplantation should be detectable in this condition, the pattern of which can be considered in a patient with progressive skin and/or lung helpful in distinguishing subtypes. disease in the presence of normal cardiac function. Diagnosis • Scleroderma renal crisis is uncommon, at around 5% of cases. It is characterized by marked hyper- reninemia, • The diagnosis of scleroderma is made on clinical grounds, so early detection of hypertension and institution of with the appearance of the hands being of greatest utility. angiotensin- converting enzyme inhibitor (ACEI) ther- • Tissue biopsy is rarely indicated. apy is vital. • Autoantibodies are detectable in the majority of cases, • Raynaud’s phenomenon and digital ulceration may be with a nucleolar- pattern ANA present in 70–90%. improved with the use of calcium- channel antagonists • Anti- centromere antibodies have sensitivity of >50% for such as felodipine or nifedipine, via vasodilatation. limited scleroderma, with a small percentage of patients • Acute digit- or limb- threatening ischemia can be treated with diff use disease also positive. Anti-centromere is with IV prostacyclin. highly specifi c for scleroderma. Patients with scleroderma frequently face great challenges • Anti- Scl- 70 antibody, whose target is DNA topoisom- with activities of daily living, chronic pain, and depression. erase 1, is more common in patients with diff use disease. Treatment aimed at providing physical and psychological • Pulmonary fi brosis can be detected on high- resolution support, and when appropriate, needs to be CT scanning and with pulmonary function tests (PFTs). part of the overall approach to management. » PFTs are the most useful tool for monitoring the Mixed connective tissue disease (MCTD) progression of lung disease, along with the 6-minute walk test. This rare condition is characterized by features of SLE, polymyositis, scleroderma and rheumatoid arthritis, in the • Echocardiography is generally used for screening for presence of autoantibodies against U1 ribonucleoprotein pulmonary arterial hypertension. (RNP). The U1 RNP antigen is actually a complex of • Endoscopy will detect the presence of esophagitis. polypeptides, a number of which can act as the epitope. The Diff erential diagnosis of skin fi ndings is outlined in ANA is usually markedly elevated in a speckled pattern. Box 16- 14. Patients present with a variety of symptoms that can develop and occur sequentially over time. Common symptoms are: Treatment • Raynaud’s phenomenon • Lung disease, and particularly pulmonary fi brosis, is the • sclerodactyly major cause of mortality in scleroderma. • hand swelling » Interstitial lung disease is the only indication for • esophageal refl ux immunosuppression in the form of high-dose cor- • myositis. ticosteroids and other agents such as cyclophospha- Less common symptoms are: mide, but results are disappointing. • pulmonary arterial hypertension • Early detection of pulmonary arterial hypertension in scleroderma patients is critical, with evidence that com- • glomerulonephritis mencement of endothelin-receptor antagonists such as • Sjögren’s syndrome bosentan, and possibly phosphodiesterase- 5 inhibitors such • skin changes as sildenafi l, can delay progression and improve prognosis. • hematological abnormalities.

538 Chapter 16 Immunology

MCTD is more common in women. It can occur, rarely, in Laboratory considerations childhood. • The presence of antiphospholipid antibodies in a serum Treatment is dictated by the nature of the organ involve- sample interferes with the activated partial thrombo- ment in the individual patient, along the lines that would be plastin time (APTT) in the laboratory, leading to pro- used for the separate disease entities. longation. This, paradoxically, gives an in vitro elevated APTT in the setting of an in vivo propensity to throm- Antiphospholipid syndrome (APS) bosis. It also renders the APTT essentially invalid for use • Antiphospholipid syndrome is characterized by: to monitor therapy with unfractionated heparin. The » recurrent fetal loss, and/or prolonged APTT is only partially correctable with mix- » recurrent venous or arterial thrombosis (Box 16-15). ing with normal serum in the laboratory, demonstrating the presence of an inhibitor. • Diagnosis is based on the presence of clinical features and, for diagnosis, persistent antibodies (>12 weeks) • Demonstration of lupus inhibitor confers an adverse against phospholipid in the form of: prognosis to the patient, with increased likelihood of » anti- cardiolipin antibodies both fetal loss and thrombosis.

» anti 2-Glycoprotein1 antibodies, or • Most laboratories measure both IgG and IgM anti- » lupus inhibitor. cardiolipin antibodies, but the IgG form is more predic- • APS occurs frequently in patients with SLE, with 25– tive of development of the clinical syndrome. 50% demonstrating autoantibodies, but an estimated • Thrombocytopenia is found in up to 25% of cases. 10–15% developing the clinical syndrome. Conversely, only 5–10% of individuals with APS also have lupus. Treatment • The etiology of the condition is unknown, but APS is • Lifelong anticoagulation is usually required for patients more common in females. with thrombotic complications of APS, with warfarin • A rapidly progressive form of the disease in which mul- the fi rst choice of drug. tiple organs can be aff ected by , including the • Direct oral anticoagulants are less eff ective than warfarin in liver and kidneys, resulting in multi-organ failure, is preventing thrombosis, particularly in high-risk patients. known as catastrophic APS. • There are no clear evidence- based guidelines for the • Antiphospholipid antibodies bind to phospholipids or patient who has antiphospholipid antibodies but is phospholipid- binding proteins on cell surfaces. This asymptomatic. Use of long-term anti-platelet therapy triggers thrombus formation within vessels, via a num- needs to be balanced against the increased risk of intra- ber of mechanisms, including: cerebral hemorrhage in this situation. » Binding of phospholipid on platelet surfaces leads • Treatment of pregnant patients with APS is dealt with in to expresssion of thrombogenic glycoprotein 2b-3a Chapter 25. and increased production of thromboxane A2. » Binding on endothelial cell and monocyte surfaces IgG4-related disease stimulates expression of adhesion molecules as well This is an increasingly recognized cluster of rare conditions, as infl ammatory and prothrombotic cytokines. characterized pathologically by swelling, fi broinfl ammatory » Phospholipid-binding proteins are anticoagulants, changes, and IgG4 plasma cell infi ltration of the aff ected so binding of these by antibody results in inhibition organs, and elevated serum IgG4 in most cases. of their function, and favors thrombosis. • Conditions now recognized to form part of this spec- Box 16-15 trum include: » autoimmune pancreatitis Clinical manifestations of APS » Mikulicz’s syndrome » retroperitoneal fi brosis • Venous thrombosis » Riedel’s thyroiditis • Arterial thrombosis » Kuttner’s tumor » Pre-term birth due to eclampsia or preeclampsia » periaortitis and periarteritis (prior to 34 weeks of gestation, fetus normal); or » mediastinal fi brosis » Fetal death beyond 10 weeks of gestation; or » infl ammatory pseudotumor » 3 or more consecutive spontaneous abortions at » eosinophilic angiocentric fi brosis <10 weeks of gestation » multifocal fi brosclerosis • Thrombocytopenia » infl ammatory aortic aneurysm • » idiopathic hypocomplementemic tubulointerstitial • Hemolysis nephritis with extensive tubulointerstitial deposits. • • Organs known to be potentially involved in this process • Neurological disease—chorea; migraine include the pancreas, periorbits, biliary tract, thyroid, • Cardiac valvular disease salivary glands, retroperitoneum, lung, breast, skin, • Renal thrombotic microangiopathy meninges, pericardium, prostate, aorta, lymph nodes, kidneys, and mediastinum.

539 Essentials of internal medicine

• The pathogenesis of this condition is unknown, but Large-vessel vasculitis there is evidence of autoimmune and allergic dysregula- tion. The IgG4 antibodies are not thought to be patho- Polymyalgia rheumatica (PMR) genic. It is more common in elderly males. • Tissue biopsy is required to make the diagnosis. At times, Clinical characteristics tissue can be replaced by tumefactive masses, making IgG4- • Polymyalgia rheumatica is a disease of unknown etiol- related disease a diff erential diagnosis for malignancy. ogy, characterized by stiff ness and aching in the limb • When recognized, the patient should be screened for girdle, particularly in the mornings. potential disease in other target organs. • Incidence increases progressively in people over the • Treatment requires immunosuppression. age of 50 years, and is more than twice as common in » Corticosteroids are the mainstay of initial treatment. females as males. » Second- line agents such as azathioprine, mycophe- • The pain centers around the shoulders in up to 95% nolate or rituximab, are oft en required for steroid- of patients, the neck and hips in the majority, and the sparing eff ect. lower back in some. • PMR causes systemic symptoms in approximately half PRIMARY VASCULITIS of those aff ected, in the form of malaise and fatigue, and can cause and night sweats. • Vasculitis can be a localized problem, or part of a sys- temic process. • Peripheral arthritis can occur. • Vasculitic disease may be primary, or occur as part of a Pathophysiology systemic disorder such as SLE or rheumatoid arthritis. • There is some evidence that the pathological lesion is a • Systemic vasculitic illnesses are generally autoimmune in low-grade synovitis, rich in CD4-positive T lympho- basis and life- threatening, requiring aggressive immu- cytes, and macrophages. nosuppressive therapy. • Vasculitis is not a feature, but PMR forms part of a The following section will focus on the primary systemic disease spectrum with giant- cell arteritis (GCA), with vasculitides. approximately 20% of PMR patients developing GCA Classifi cation of the vasculitides is confusing. Some and around 50% of GCA patients also suff ering PMR. authors classify according to the size of the vessels involved, while others recommend groupings based on association Diagnosis with anti-neutrophil cytoplasmic antibodies (ANCAs). Box • PMR is invariably accompanied by an acute phase 16- 16 groups the illnesses taking both features into account. response, resulting in elevated ESR, CRP, and at times More recently, a revised, more comprehensive classifi - abnormal liver function tests and a normocytic anemia, cation system has been recommended (Box 16- 17). This although there are cases described with a normal ESR. system will be used to discuss the diseases addressed below. • MRI or bone scanning may demonstrate synovitis in proximal joints. • The diagnosis is clinical. Box 16-16 Treatment Classifi cation of vasculitides • Failure of symptoms to resolve with low- dose corti- Large-vessel, non-ANCA-associated costeroids, usually with a starting dose of 15 mg/day, should prompt reconsideration of the diagnosis. • Giant-cell arteritis/Polymyalgia rheumatica • Patients may need long- term treatment, or the disease • Takayasu arteritis may eventually spontaneously remit. Medium-vessel, non-ANCA-associated • Patients should be counseled to seek urgent review if • they develop symptoms suggestive of GCA, such as • Kawasaki disease headache or visual disturbance.

Small-vessel, ANCA-associated Giant-cell arteritis (GCA) • Granulomatosis with polyangiitis (GPA, previously Wegener’s granulomatosis) • GCA is a life- threatening vasculitis involving large and medium-sized muscular arteries, especially those of • Eosinophilic granulomatosis with polyangiitis (EGPA, previously Churg-Strauss vasculitis) the external carotid system. GCA typically involves the temporal arteries—hence its alternative, but less accu- • Microscopic polyarteritis rate, name ‘temporal arteritis’. Small-vessel, non-ANCA-associated Clinical characteristics • Leukocytoclastic vasculitis • Patients are usually elderly. • Essential (see Chapter 9) • Henoch–Schönlein purpura • Atherosclerosis, heavy smoking, and the HLA- DRB1 haplotype are risk factors.

540 Chapter 16 Immunology

Box 16-17 Names for vasculitides adopted by the 2012 International Chapel Hill Consensus Conference on the Nomenclature of Vasculitides

Large-vessel vasculitis (LVV) Single-organ vasculitis (SOV) • Takayasu arteritis (TAK) • Cutaneous leukocytoclastic angiitis • Giant-cell arteritis (GCA) • Cutaneous arteritis Medium-vessel vasculitis (MVV) • Primary central vasculitis • Polyarteritis nodosa (PAN) • Isolated aortitis • Kawasaki disease (KD) • Others Small-vessel vasculitis (SVV) Vasculitis associated with systemic disease • Anti-neutrophil cytoplasmic antibody (ANCA)–associated • Lupus vasculitis vasculitis (AAV) • Rheumatoid vasculitis • Microscopic polyangiitis (MPA) • Sarcoid vasculitis • Granulomatosis with polyangiitis (Wegener’s) (GPA) • Others • Eosinophilic granulomatosis with polyangiitis Vasculitis associated with probable etiology (Churg–Strauss syndrome) (EGPA) • virus–associated cryoglobulinemic vasculitis • Immune complex SVV • virus–associated vasculitis • Anti–glomerular basement membrane (anti-GBM) disease • Syphilis-associated aortitis • Cryoglobulinemic vasculitis (CV) • Drug-associated immune complex vasculitis • IgA vasculitis (Henoch–Schönlein) (IgAV) • Drug-associated ANCA-associated vasculitis • Hypocomplementemic urticarial vasculitis (HUV) • Cancer-associated vasculitis (anti-C1q vasculitis) • Others Variable-vessel vasculitis (VVV) • Behçet’s disease (BD) • Cogan’s syndrome (CS)

Jennette JC et al. 2012 Revised International Chapel Hill Consensus Conference Nomenclature of Vasculitides. Arthritis Rheumatism 2013;65(1):1–11.

• Patients present with severe headache, scalp tenderness, • Diagnosis is supported by temporal artery biopsy reveal- malaise, fevers, and weight loss. ing granulomatous infl ammation of the full thickness • Focal neurological symptoms can occur, depending on of the arterial wall, with infi ltration by multinucleated pattern of vessel involvement. giant cells, lymphocytes and fi broblasts. The intima is • Jaw claudication with chewing, and tongue claudication oft en markedly hyperplastic, with stenosis and occlu- are not pathognomonic, but have a high positive predic- sion of the vessel. False negatives can occur due to ‘skip’ tive value for this condition. lesions, and biopsies should be taken within 1 week of the commencement of corticosteroids. A negative result • Aortic arch syndrome and neuropathy occur in a does not exclude the diagnosis, and treatment should minority. not be delayed for a biopsy to be performed. • Physical examination will usually reveal fever, and • Imaging techniques of the temporal arteries such as with marked tenderness over the scalp with thickened, tender ultrasound or MRI do not yet have a defi ned role. temporal arteries which may have lost their pulse. • Visual symptoms are a medical emergency, as they Treatment indicate embolization or vasculitis of the ophthalmic artery, which is a branch of the external carotid. Sudden • High- dose corticosteroids need to be administered blindness may result. immediately upon suspicion of the diagnosis of GCA, either orally (≥1 mg/kg/day of prednisolone) or intra- Investigations venously (methylprednisolone 500 mg daily for 3 days • Laboratory investigations reveal markedly elevated ESR followed by high- dose oral prednisolone). and CRP, and oft en a normocytic anemia, thrombo- • Once clinical and serological parameters return to normal, cytosis, and abnormalities on liver function tests. gradual dosage tapering can occur with careful monitoring.

541 Essentials of internal medicine

• If unacceptably high doses of steroids are required for • Approximately 30% of patients will be hepatitis B disease control, a second immunosuppressive agent surface-antigen positive; it is important to identify such as cyclophosphamide or methotrexate may be patients with active hepatitis B infection, as the treat- required. ment will diff er despite the disease being identical. • There is growing evidence for the use of IL-6 receptor • PAN has also been associated with other infections such antagonist tocilizumab in this condition. as HIV, hepatitis C and (CMV), with Many patients will go into apparent long-term remission autoimmune disease such as rheumatoid arthritis, and aft er a period of immunosuppression, but approximately with malignancy e.g. Hodgkin’s lymphoma and hairy- 50% of patients suff er a fl are with steroid reduction. cell leukemia. • PAN is more common in men. Takayasu arteritis Most patients have marked constitutional symptoms of Takayasu arteritis is a disease that mainly aff ects young fever, night sweats, weight loss, fatigue, and malaise. More females, with greatest prevalence in Asia. specifi c symptoms and signs (Box 16-18) will generally be • It is a granulomatous vasculitis of the proximal aorta and due to organ ischemia. its major branches. Diagnosis and treatment • Fibroproliferative changes are found in the intima of the • There is no specifi c diagnostic test for PAN, other than aff ected vessels, resulting in variable stenosis, thrombo- demonstration of necrotizing vasculitis on tissue biopsy. sis, and aneurysmal dilatation. Testicular biopsy is relatively safe, and can be diagnostic Clinical presentation is invariably with ischemia of the in patients with testicular symptoms. aff ected region. It is oft en referred to as ‘pulseless disease’. • Imaging may demonstrate irregularities of vessels such Patients may present with: as stenosis or aneurysm formation. • transient ischemic attacks • Treatment is with immunosuppression, generally com- • mencing with high-dose corticosteroids and then with a • limb claudication second agent such as cyclophosphamide. • visceral ischemia • Patients with hepatitis B should be treated with antiviral • hypertension that can result from renal artery stenosis medication. • constitutional symptoms of fever, malaise, and weight Kawasaki disease loss (frequent) • Kawasaki disease is a disease almost exclusively of child- • serological evidence of an acute- phase response hood that commences with high fever and causes sys- • absent peripheral pulses temic vasculitis of medium- sized arteries. • vascular bruits • The acute phase of the illness is characterized by high fever • aortic regurgitation due to dilatation of the aortic root. which lasts for upward of a week. The more prolonged Imaging with MRI and CT angiography is a mainstay in the fever, the greater the risk of cardiac complications. the diagnosis of the disease. PET scan is increasingly used, • Kawasaki disease is common in East Asia. Up to 90% of but its role in diagnosis remains unclear. cases occur before age 5. The etiology is unknown. Treatment consists of: • Immunosuppression for the vasculitis. Box 16-18 • Corticosteroids in high doses are used for initial remis- sion induction. Clinical features in patients with • Second agents such as methotrexate, cyclophosphamide polyarteritis nodosa or mycophenolate may be needed for remission main- • Angina pectoris tenance. Biological agents such as anti-TNF antibodies • are showing promise in this condition. • Mesenteric angina • Surgical methods to restore blood fl ow in occluded • Intestinal infarction vessels where possible. » Surgical approaches depend on the location and • Peripheral neuropathy severity of lesions, and range from percutaneous • Mononeuritis multiplex angioplasty through to stenting and bypass. • Stroke • Cutaneous vasculitis Medium-vessel vasculitis • Asymmetric polyarthritis • Testicular pain Polyarteritis nodosa (PAN) • • Polyarteritis nodosa is a rare vasculitis of unknown etiol- • Hypertension ogy aff ecting medium to small arteries. The disease has • Renal infarction a propensity for mesenteric, renal and coronary arteries.

542 Chapter 16 Immunology

• GPA can be a fulminant, life- threatening disease, and Box 16-19 lung involvement is frequently misdiagnosed as infec- Clinical manifestations of tion in the fi rst instance, resulting in considerable peril Kawasaki disease for the patient. Common Less common CLINICAL PEARL • Prolonged fever • Arthralgia • Cervical • Arthritis Granulomatosis with polyangiitis should be a diff er- lymphadenopathy ential diagnosis in any patient with a pulmonary–renal • Diarrhea syndrome, , or respiratory tract symptoms • Mucocutaneous • Myocarditis/pericarditis that fail to respond to antibiotics. Delays in diagnosis lesions: conjunctivitis, • Aseptic signifi cantly worsen both the pulmonary and the renal diff use maculopapular prognosis. rash, • Pneumonia tongue, infl amed lips, • Hepatitis pharyngitis, edema of Box 16- 20 lists the clinical manifestations of GPA. the palms and soles, • The majority of patients present with upper respiratory desquamation of the tract symptoms of variable duration and severity, prior fi ngers to the development of systemic constitutional features and those related to involvement of various organs. • Cavitating lung lesions are typical, with or without • The clinical abnormalities (Box 16- 19) are due to vas- interstitial changes. culitis, and this is also responsible for the feared compli- cation of aneurysmal dilatation of arteries, especially the • The renal lesion is a pauci-immune, focal necrotizing coronary arteries. This in turn can lead to rupture or glomerulonephritis. thrombosis of the vessel, leading to myocardial ischemia • Development of renal failure can be rapid. or infarction. Aneurysm formation generally occurs between 2 and 12 weeks of disease onset. Diagnosis • The detection of a c-ANCA (cytoplasmic pattern Diagnosis and treatment ANCA) with anti- PR3 (proteinase 3) antibodies in a • There is no specifi c diagnostic test for Kawasaki disease, patient with consistent clinical features is diagnostic of so clinical features are key. GPA. • A high index of suspicion is necessary, given the impor- » Rarely, p-ANCA (perinuclear pattern ANCA) tance of instituting treatment promptly to lessen the risk is found in patients with the more typical clinical of aneurysm formation. phenotype of GPA than microscopic polyangiitis (which is described below). • Most patients display an acute phase response. » In ANCA- negative patients, tissue diagnosis will • High- dose IV immunoglobulin should be administered be required. This will usually be in the setting of as soon as the diagnosis is made. If fever does not resolve sino nasal disease, so biopsy of the nasal mucosa may promptly, a second dose should be given. reveal vasculitis. • Aspirin also improves prognosis and is used in low doses to reduce the risk of thrombosis. Box 16-20 CLINICAL PEARL Clinical manifestations of GPA Aspirin is generally contraindicated in children because of the risk of Reye’s syndrome. Kawasaki disease is one Rhinosinusitis of the rare exceptions to this rule, as benefi t outweighs Epistaxis risk in this case. Cough Dyspnea Small-vessel vasculitis Dysphonia Hemoptysis Granulomatosis with polyangiitis (GPA)/ Wegener’s granulomatosis (WG) Fever • Granulomatosis with polyangiitis is a Renal failure with a propensity to involve the and Hypertension the kidneys. Arthralgia • It is one of the ANCA-associated vasculitides, with up Headache to 90% of suff erers ANCA-positive, especially those Focal neurological symptoms with systemic rather than localized disease.

543 Essentials of internal medicine

• While tissue biopsy will reveal granulomatous necro- Diagnosis and treatment tizing vasculitis, this may not always be possible in the • MPA is characterized by p- ANCA (perinuclear ANCA) setting of an acutely unwell patient. Treatment needs and the presence of antibodies to MPO (myeloperoxi- to be instituted immediately when respiratory, renal or dase) in about 80% of cases. neurological involvement is present. » Occasionally, c- ANCA is detected in patients who • There is usually serological evidence of an acute phase behave clinically more like MPA than GPA. reaction. • The presence of a positive p- ANCA and MPO antibod- • Microscopic urine examination, determination of renal ies has high positive predictive value for the diagnosis function, and lung imaging is essential in all patients. of MPA. Renal biopsy is desirable, however, for both It is worthwhile to obtain CT imaging of the paranasal defi nitive diagnosis and prognostic reasons. sinuses. • Determination of renal and pulmonary function at base- • Baseline testing of pulmonary function should be line is essential, as is lung imaging. performed. The treatment approach is similar to that described for GPA. Treatment • Without aggressive immunosuppression, the prognosis Eosinophilic granulomatosis with polyangiitis of GPA is dismal. (EGPA)/Churg–Strauss syndrome (CSS) • The use of cyclophosphamide has resulted in a 5-year Clinical characteristics survival rate of >90%, although progression to end- stage kidney disease remains a risk. • Eosinophilic granulomatosis with polyangiitis is a rare eosinophilic vasculitis of small vessels that arises in • High-dose IV corticosteroids should be instituted patients with a history of atopic disease. Most patients immediately for any patient with lung or kidney disease, will have a history, oft en for many years, of allergic concurrently with cyclophosphamide or rituximab. rhino sinusitis and asthma. » Some patients may be able to be maintained on methotrexate or mycophenolate, given the signifi - • When vasculitis develops, patients present with a com- cant long- term toxicity associated with cumulative bination of blood eosinophilia and pulmonary infi l- doses of cyclophosphamide. trates, sometimes with hemoptysis, vasculitic skin » It is recommended that patients on long- term rash, and neurological features in the form of either immunosuppression be treated with prophylaxis symmetrical peripheral neuropathy or mononeuritis for Pneumocystis jirovecii in the form of low- dose oral multiplex. - sulfamethoxazole. Diagnosis • The aim of treatment is to achieve remission. This can be assessed by clearing of respiratory tract lesions, and • Around half of patients are p- ANCA- positive. normalization or stabilization of renal function. • Tissue biopsy of skin or sural nerve may be helpful for • The titer of c- ANCA and PR3 antibodies may be a diagnosis if cutaneous or neurological disease is present. guide to disease activity, and can be used for long-term An eosinophilic vasculitis is diagnostic. monitoring of patients. • Lung biopsy is generally not needed when characteristic • Occasionally, fulminant disease requires plasma radiological abnormalities are present in an atopic patient. exchange. • Bronchoalveolar lavage will reveal - rich fl uid if performed.

Microscopic polyangiitis (MPA) Treatment Similar to GPA, MPA is a small-vessel vasculitis associated As is typical for most eosinophilic disorders, corticoster- with ANCA. oids are eff ective, however, some patients require ongo- ing therapy with unacceptably high doses. For those with Clinical characteristics severe, multi-organ disease, cyclophosphamide can be used • Patients with MPA generally present with constitutional to induce remission. A second agent such as azathioprine symptoms, and renal impairment with active urinary or mycophenolate may be required to maintain remission sediment. or to allow reduction of the corticosteroid dose. Anti-IL-5 • Lung disease will oft en cause hemoptysis secondary monoclonal antibodies, such as mepolizumab also appear to to pulmonary vasculitis. be eff ective in this condition. • Upper airway involvement is not usual, in contrast to GPA. IgA vasculitis (IgAV)/Henoch-Schönlein • Skin, joint, and neurological complications aff ect a purpura (HSP) minority. IgA vasculitis is a systemic vasculitis which is much more • The ratio of lung:renal involvement is reversed com- common in children than adults. The disease results from pared with GPA, whereby renal involvement in MPA is deposition of IgA- containing immune complexes in vessel present in 90% of patients. walls, stimulating an infl ammatory response.

544 Chapter 16 Immunology

Clinical characteristics • Severe episodes, and those with renal impairment or • of the lower extremities is invariable, evidence of organ failure, require treatment with sys- very oft en extending up to the buttocks (Figure 16- 15). temic corticosteroids and possibly second-line agents such as cyclophosphamide or mycophenolate. • Joint involvement and abdominal pain occur in the majority of cases. » The arthritis mainly aff ects the joints of the lower Single-organ vasculitis limbs. » The abdominal pain may be accompanied by bloody Cutaneous leukocytoclastic angiitis diarrhea. This may occur in isolation, or in association with a systemic • Around 40% of suff erers will have microscopic vasculitis or autoimmune disease (Box 16- 21). hematuria. • Drug reactions are a common cause when there is no • Renal impairment is rare, but a small number of associated systemic illness. patients, usually adults, will progress to end-stage renal • Reactions to vaccines or foods are also a possibility. disease. • Sometimes the disease appears to be idiopathic. • IgAV usually follows on from infection, mostly of the • Systemic diseases include SLE, SS, IgAV, rheumatoid upper respiratory tract. Both Streptococcus and Staphylo- arthritis (RA), and cryoglobulinemia, with the patho- coccus have been implicated in some case series. genetic mechanism being immune- complex deposition. Diagnosis Clinically, patients have tender, non-blanching purpuric or petechial lesions which nearly always start on the distal lower • Diagnosis is generally obvious when faced with the limbs, but may spread proximally. Biopsy reveals perivas- combination of the lower extremity purpura, and joint, cular neutrophils with fi brinoid necrosis (leukocytoclasis; GI tract or renal disease. Figure 16- 16, overleaf ). • Renal biopsy changes are identical to IgA nephropathy • The rash is self- limited once the inciting antigen (e.g. when the kidneys are involved. antibiotic) is withdrawn. Treatment • When associated with systemic disease, treatment is according to the usual protocols for that disease. • Most episodes resolve without treatment within 3 weeks. Variable-vessel vasculitis Behçet’s disease • Behçet’s disease is a rare disorder characterized by pain- ful oral (Figure 16- 17, overleaf ) and genital aphthous ulceration, uveitis, and vasculitis that can aff ect multiple organs, including the central nervous system, retina and skin.

Box 16-21 Diff erential diagnosis of palpable purpura Leukocytoclastic vasculitis (angiitis) • Idiopathic • Cryoglobulinemia • Malignancy • IgA vasculitis (Henoch-Schönlein purpura) • Malignancy • Systemic autoimmune disease—systemic lupus erythematosus, Sjögren’s syndrome, rheumatoid arthritis Cutaneous emboli Infection • Meningococcus Figure 16-15 The typical palpable purpura seen in Henoch-Schönlein purpura (IgA vasculitis) • Gonococcus • Rickettsia (e.g. Rocky Mountain spotted fever) From Habif TB. Clinical dermatology, 5th ed. St Louis: Elsevier, 2009.

545 Essentials of internal medicine

• Vasculitic complications require immunosuppression with corticosteroids and oft en second- line agents. • Ocular disease especially needs an aggressive approach, with anti- TNF monoclonal therapy gaining favor in this situation.

AUTOINFLAMMATORY DISORDERS These are a group of monogenic syndromes that result in infl ammasome dysregulation and overproduction of pro- infl ammatory cytokines, especially IL-1-beta. Characteristic features include fever, rash, and serositis. Mostly very rare, there are now a large number of identifi ed syndromes, but Figure 16-16 This photomicrograph shows Familial Mediterranean fever and TNF-receptor- associated leukocytoclasis, with evidence of fi brinoid necrosis and periodic syndrome are the most prevalent of these. polymorphonuclear cells surrounding the blood vessel From Elston D et al. Dermatopathology. Philadelphia: Elsevier, 2008. Familial Mediterranean fever (FMF) Clinical characteristics This most common of the autoinfl ammatory syndromes aff ects predominantly those of ethnicity from the popula- tions located around the Mediterranean Sea. FMF results from a mutation in the MEFV gene, and is autosomal recessive. • Most patients present in childhood or adolescence. • The disease is characterized by recurrent acute attacks of peritonitis, high fever, and a typically large- joint monoarthritis. • Pleuritis, pericarditis, and testicular pain can also occur, as can an intensely erythematous rash that mainly aff ects the lower limbs. • Attacks occur at variable frequency. In severe cases they may be weekly. Figure 16-17 Oral ulcer in Behçet’s disease • Many patients undergo abdominal surgery due to uncertainty regarding the cause of peritonitis. From Krachmer JH, Mannis MJ and Holland EH. Cornea, 3rd ed. St Louis: Elsevier, 2010. • Those with frequent attacks over a prolonged period are at risk of the development of secondary amyloidosis as a result of prolonged elevation in levels of the acute phase • An oligoarthritis, recurrent abdominal pain, and pul- reactant serum amyloid A protein. Renal amyloidosis is monary vasculitis rarely may occur. the most common site. • More common in women, there is a striking racial dis- position with Behçet’s mainly aff ecting those from Iran, Diagnosis Turkey and East Asia. • Behçet’s does not result in autoantibody formation, • Infl ammatory markers are raised during an attack. ESR, but will usually be accompanied by an acute phase CRP, white cell count, and platelets are usually mark- response. edly elevated. • The skin of most patients with Behçet’s displays path- • These parameters usually normalize between attacks. ergy—puncturing the skin with a needle results in the • The defi nitive test is demonstration of the abnormal formation of a pustule aft er 24–48 hours. MEFV gene.

Treatment Treatment • Recurrent ulceration may be ameliorated by treatment • Long-term administration of at a dose of with colchicine. 0.5–1.5 mg daily reduces the number and severity • Both dapsone and have been reported to of attacks, and the risk of amyloidosis, in the majority display effi cacy. of patients.

546 Chapter 16 Immunology

• Interleukin-1 (IL-1) inhibition with monoclonal anti- Table 16-14 Primary and secondary immunodefi ciency body therapy (anakinra, , ) is safe and eff ective in those unresponsive to or intolerant TYPE EXAMPLE(S) of colchicine. Primary immunodefi ciency TNF-receptor-associated periodic Predominant antibody Common variable syndrome (TRAPS) defi ciencies immunodefi ciency Less common than FMF, TRAPS is an autosomal domi- Phagocyte defects Chronic granulomatous disease nant condition resulting from a mutation in the TNFRFS Combined T and B Severe combined 1A gene which encodes the TNF (tumor necrosis factor) cell defi ciencies immunodefi ciency receptor. Combined T and B DiGeorge syndrome Clinical characteristics cell defi ciencies with syndromic features • Onset is generally in early childhood (<10 years) but can Secondary immunodefi ciency be delayed until adulthood. • Fever oft en lasts for 10–14 days. Environmental Malnutrition • Abdominal pain is typical, and oft en severe. Infection Human immunodefi ciency virus • Myalgia and arthralgia occur frequently, along with Disease-related Malignancy; protein-losing headache, pleuritis, a variety of skin rashes, and ocular enteropathy; burns; widespread infl ammation in the form of conjunctivitis or uveitis. skin disease • Secondary amyloidosis can be a chronic complication. Iatrogenic Cancer chemotherapy; Immunosuppressive drugs Diagnosis • Infl ammatory markers are raised during an attack. ESR, CLINICAL PEARL CRP, white cell count, and platelets are usually ele- vated, oft en markedly so. These do not always return to Immunodefi ciency should be suspected in the setting of: • recurrent or chronic infection normal between attacks. • infection with low-virulence organisms • Defi nitive diagnosis depends on detection of the abnor- • evidence of end-organ damage, e.g. bronchiectasis mal TNFRFS 1A gene. • poor response to standard antimicrobial therapy • recurrent infection with the same organism. Treatment • IL-1 inhibition is the mainstay of treatment in this Primary immunodefi ciency disorder. The true incidence of primary immunodefi ciency is • TNF blockade with etanercept is eff ective or partially unknown, but is relatively common if selective IgA defi - eff ective in some patients. ciency is taken into account. Approximately 65% of cases are antibody defi ciencies. Unfortunately, many cases are missed worldwide; however, neonatal screening for some IMMUNODEFICIENCY conditions has been implemented in certain countries. • Diagnosis is frequently delayed for an average of over Defi ciencies of the immune system are common, and can 5 years. be divided into primary (congenital) and secondary (acquired). • Given that earlier diagnosis improves prognosis, many • Primary immunodefi ciency refers to a defective com- patients will have developed complications by the time ponent, or components, of the immune system due to a their underlying immunodefi ciency is recognized. genetic mutation. • Consanguinity is the major risk factor for primary • Secondary immunodefi ciency occurs as a result of a immunodefi ciency. wide array of diseases, and their treatments. Box 16-22 (overleaf ) outlines clinical clues to diagnosis at Table 16- 14 gives examples of defi ciencies of each type. diff erent ages. For the clinician, the challenge will oft en be: • to recognize when a patient with infection is immuno- CLINICAL PEARL defi cient, and thus be in a position to adapt treatment Infection is the major complication of immunodefi - appropriately ciency, but it is important to remember that immunode- fi ciency is also associated with immune dysregulation. • to look at ways to protect the patient from future risk, Some patients will develop autoimmunity, allergy and for example through prophylaxis, replacement therapy, malignancy as a result of their immunodefi ciency. or immunization.

547 Essentials of internal medicine

Box 16-22 When to consider a diagnosis of primary immunodefi ciency Infancy Childhood Any age • • 8 or more infections in 1 year, • , e.g. with • Family history of primary especially ear infections Pneumocystis jirovecii immunodefi ciency • Persistent candidiasis • Recurrent pneumonia or sinusitis • Omphalitis • Recurrent skin infection • Evidence of end-organ damage, • Congenital defects • Recurrent e.g. bronchiectasis • Consanguinous parents • Failure of response to standard antibiotics • Autoimmune disease

Primary antibody defects • paranasal sinus imaging should be performed if patients Antibody defects generally manifest with recurrent sinopul- are symptomatic monary infection in the form of otitis media, sinusitis and • patients are unlikely to have an adequate response to pneumonia (Table 16- 15). Frequent, recurrent, and chronic most immunization infection should raise suspicion. • hyperimmune globulin should be used where available if exposure to a relevant infection is reasonably suspected. Investigating antibody defi ciency The initial investigation of a patient with suspected primary Phagocyte defi ciency antibody defi ciency should be with: Phagocyte defi ciency accounts for approximately 10% of • full blood count primary immunodefi ciency. Most defi ciency in phagocyte • serum immunoglobulins number or function is acquired, secondary to infection, • specifi c antibody responses following vaccination with medication, nutrition or malignancy, with the primary protein antigens (tetanus, pertussis), and polysaccharide forms being due to rare genetic defects. antigens (Pneumococcus). If these are normal: Primary neutropenia • investigate for complement defi ciency: CH50, AH50, and severe congenital neutropenia are C2, C4, C3. rare conditions with defi ned genetic mutations. If there is evidence of lymphopenia and/or hypogamma- globulinemia, follow- up testing should be sought with: Combined antibody and cellular • lymphocyte subsets immunodefi ciencies • specifi c gene testing. Congenital T- cell defi ciency has a profound eff ect on the development and function of B cells also, and hence patients present with problems related to both cell-mediated and CLINICAL PEARL humoral immunodefi ciency. Diff erential diagnosis in recurrent sinopulmonary infec- Patients generally present as infants, and without treat- tion: ment do not survive. • primary or secondary immunodefi ciency • allergic rhinosinusitis Severe combined immunodefi ciency • chronic rhinosinusitis with or without polyposis • cystic fi brosis • Rare; approximately 1:65,000. • ciliary dyskinesia • Consanguinity is a major risk factor. • anatomical sinonasal obstruction. • Infants present with , failure to thrive, and oppor- tunistic infections. Long-term management of antibody • Multiple gene defects have been identifi ed, e.g. adenos- defi ciency ine deaminase defi ciency, JAK3 defi ciency. Patients who are on long- term immunoglobulin replace- • Treatment involves stem- cell transplantation or gene ment therapy should be kept in regular review: therapy. • IgG levels should be monitored to ensure adequate replacement Chronic granulomatous disease (CGD) • lung function testing and imaging should be performed • This is a rare condition with an incidence of approxi- at regular intervals to screen for bronchiectasis mately 1:250,000.

548 Chapter 16 Immunology

Table 16-15 Primary antibody defi ciencies

PRIMARY IM- MUNODEFI- GENETIC CLINICAL CIENCY INCIDENCE DEFECT FEATURES DIAGNOSIS TREATMENT

Selective IgA Common; Unknown 80% asymptomatic No measurable Antibiotic therapy defi ciency 1:500 to 1:700 Recurrent IgA as required sinopulmonary infection Celiac disease Autoimmune disease

Common variable 1:25,000 to Mutations Onset may be delayed Low IgG Regular immunodefi ciency 1:50,000 defi ned in until teenage years or Low IgM and IgA replacement (CVID) about 15%; young adulthood is usual therapy with includes TACI, Recurrent bacterial pooled donor ICOS genes Poor or absent immunoglobulin infection, especially specifi c antibody sinopulmonary; response to septicemia, abscesses vaccination Bronchiectasis Autoimmune disease

Specifi c antibody Unknown Unknown Recurrent bacterial Absent or sub- Regular defi ciency infection, especially normal response replacement sinopulmonary; otitis to vaccination therapy with media (e.g. pneumo- pooled donor coccal, tetanus) immunoglobulin

IgG sub-class Unknown Unknown Clinical signifi cance Low levels of Nil defi ciency unclear one or more No longer IgG sub-classes recommended in presence of in screening for normal total IgG immunodefi ciency

X-linked agamma- 1:70,000 to BTK gene Almost exclusively Absent B cells Regular globulinemia 1:400,000 seen in boys; presents Low or absent replacement with in childhood with IgG, IgM, IgA pooled donor recurrent bacterial immunoglobulin infection BTK gene abnormality

BTK, Bruton’s tyrosine kinase; ICOS, inducible co-stimulator; TACI, transmembrane activator and calcium-modulator and -ligand interactor.

• Patients with CGD tend to present with recurrent deep- Combined antibody and cellular seated infections due to Staphylococcus aureus, Burkholde- immunodefi ciencies with syndromic features ria cepacia, Serratia marcescens, Nocardia spp. and Aspergillus These are outlined in Table 16-16 (overleaf ). spp. Hepatic and pulmonary infections are frequent. • CGD is caused by mutation in one of the fi ve genes that CLINICAL PEARL encode the nicotinamide adenine dinucleotide phos- phate (NADPH) phagocyte oxidase complex (Phox) Relatively few pathology tests need to be employed in which is responsible for the that kills the initial screening of a patient with suspected primary phagocytosed bacteria in neutrophils. immunodefi ciency: • full blood count and fi lm • Diagnosis is by demonstration of reduced phagocyte • serum immunoglobulins oxidase activity, either by a negative NBT (nitroblue • complement levels tetrazolium) test by or a reduction in the dihydro- • specifi c antibody responses. rhodamine 123 assay. If all of these are normal, primary immunodefi ciency is unlikely. If the lymphocyte count is low, specifi c B- • Treatment is with long- term administration of gamma- and T-cell subsets should be measured. If Howell–Jolly interferon, prophylactic antibiotics (trimethoprim- bodies are present on blood fi lm, is likely. sulfamethoxazole) and antifungals (itraconazole).

549 Essentials of internal medicine

Table 16-16 Examples of primary combined immunodefi ciencies with syndromic features

T-CELL IMMUNO- GENE DEFICIENCY INCIDENCE DEFECT CLINICAL FEATURES DIAGNOSIS

DiGeorge syndrome 1:4,000 Deletion of Typical facies Reduction in T-cell chromo some (Figure 16-18) numbers 22q11.2 Recurrent infections Decreased antibody Hypocalcemia due to responses to protein parathyroid hypoplasia antigens Schizophrenia Developmental delay Cardiac anomalies

Ataxia-telangiectasia 1:100,000 ATM Progressive ataxia Low Ig levels Ocular and cutaneous B and T cell lymphopenia telangiectasia from age Poor antibody responses 3–5 years Elevated alpha-fetoprotein Recurrent bacterial infections

Wiskott–Aldrich 1:100,000 to WASP Severe eczema; recurrent Thrombocytopenia with syndrome 1:1,000,000 X-linked recessive bacterial infection, small platelets frequently staphylococcal Decreased T cells Low Ig levels

Hyper-IgE 1:100,000 STAT3 Eczema; skin abscesses Very high total serum IgE (Figure 16-19) Eosinophilia Recurrent staphylococcal infection Pneumatoceles Candidiasis Typical facies Retention of primary teeth (Figure 16-19)

Ig, immunoglobulin.

Vaccination in PID secondary to poor living conditions, or communicable dis- ease. In the developed world, aging of the population with • Live viral and bacterial vaccines should not be given. the accompanying immunosenescence accounts for a sub- » Specifi cally, oral polio vaccine, bacille Calmette- stantial proportion of the increased susceptibility to infec- Guérin (BCG), measles- mumps- rubella (MMR) tion, along with immunosuppressive treatments used in the vaccine, yellow fever, live typhoid, varicella vaccine, treatment of a variety of malignant and autoimmune condi- live infl uenza vaccine, and live oral rotavirus vaccine should be avoided. tions, and in the management of transplanted organs. • Inactivated and sub-unit vaccines are safe, but may be ineff ective; antibody response should be measured to HIV/AIDS check effi cacy. • Passive immunization should be considered where The human immunodefi ciency virus (HIV) pandemic has appropriate, e.g. VZV (varicella zoster virus) presented unique global challenges both clinically and eth- immunoglobulin. ically. The acquired immune defi ciency syndrome (AIDS) has aff ected all nations and all sections of society. There have been deep insights into the understanding of both the Secondary (acquired) intact and the disordered immune system as a result of this immunodefi ciency disease. A previously nearly universally fatal condition has The majority of cases of secondary immunodefi ciency turned into a chronic, treatable condition due to the advent globally result from poor nutrition and chronic infection of eff ective antiretroviral therapy.

550 Chapter 16 Immunology

Figure 16-18 Patients with typical facies for DiGeorge syndrome Figure 16-19 Patient with hyper-IgE syndrome, From Fung WLA et al. Extracardiac features predicting 22q11.2 demonstrating severe atopic dermatitis (above) and deletion syndrome in adult congenital heart disease. Int J Cardiol retention of primary teeth (below) 2007;131(1):51–58. From Esposito L et al. Hyper-IgE syndrome: dental implications. Oral Surg Oral Med Oral Pathol Oral Radiol 2012;114(2):147–53.

Epidemiology Other risk factors include: UNAIDS, the Joint United Nations Programme on HIV/ • concurrent genital infection AIDS, estimates that in 2019: • uncircumcised males • 36.9 million people are infected with HIV globally • transfusion of infected blood products • 21.7 million people were accessing antiretroviral therapy • exposure from medical or dental procedures. • 940,000 people died of the disease • 1.8 million people were newly infected, but new infec- Pathophysiology tions were reduced by 47% since the peak in 1996 • HIV is transmitted through blood, genital fl uids or • sub-Saharan Africa accounts for the majority of people breast milk. living with the infection » 79% of new infections in 10- to 19-year-olds were • HIV gains entry to cells through the CD4 molecule in females found on the surface of CD4+ cells, T-helper lympho- cytes, monocyte-macrophages, and dendritic cells. • high rates of infection are found in South America, Asia » The CD4 molecule acts as the receptor for an HIV and Eastern Europe. envelope protein known as gp120. » For successful entry into the cell, HIV gp120 must Risk factors for HIV infection also bind a co- receptor, which is usually a protein in • Heterosexual transmission remains the predominant a class known as chemokine receptors. mode of acquisition of the virus globally. » The two major chemokine receptors used by HIV • Male- to- male sex carries the highest risk in devel- for cell entry are CCR5 and/or CXCR4. Diff er- oped Western countries and is an important risk factor ences in gp120 protein structure determine tropism CCR5 blockade elsewhere. for either of these co- receptors. is one therapeutic target in HIV treatment. Other key risk groups include: • To enter the cell, the virus undergoes fusion with the mem- • children, through perinatal transmission brane, a step that can be blocked by fusion inhibitors. • people who inject drugs • HIV is a single-stranded RNA retrovirus that requires • sex workers. reverse transcription in the cell cytoplasm, through the

551 Essentials of internal medicine

action of the enzyme reverse transcriptase, to form single- • Severe acute illness with high levels of viremia is poorly stranded and then double- stranded DNA. This step can prognostic. be blocked by reverse transcriptase inhibitors. • Highly-sensitive enzyme immunoassays are used as • For the newly formed HIV DNA to be copied, it must screening tests, but confi rmation of HIV infection next be integrated into the host cell genome via the requires a positive Western blot test. Recommended action of the enzyme integrase. This is the next target baseline investigations are outlined in Box 16- 24. for therapeutic blocking of the process, with the use of • The formation of HIV antibodies signals the end of the integrase inhibitors. seroconversion phase, but antibodies—although detect- • Following successful transcription, the newly formed able at 3 months in the vast majority—may occasion- viral polypeptides are spliced by the enzyme protease ally take up to 6 months or even longer to be detectable into their functional forms, a step susceptible to the in serum. For this reason, earlier diagnosis can require action of protease inhibitors. detection of viral antigen in the serum, with p24 antigen • The viral particles and structural proteins are then testing or nucleic acid amplifi cation testing. assembled in the cytoplasm into mature virus which is ready for release. Chronic HIV infection Most individuals mount a prompt and vigorous immune • Untreated, the majority of patients decline physically response to acute HIV infection, via formation of antibody and immunologically aft er a variable-length clinically and the action of cytotoxic T cells and natural killer cells. latent period. • A high viral load and a sudden drop in CD4+ T- cell num- • Autoimmune phenomena may be seen prior to bers during the acute infection phase is typically seen, fol- the development of signifi cant immunodefi ciency lowed by restoration as the immune response kicks in. (Box 16-25). This is oft en associated with a CD8+ lym- • The virus is directly cytopathic to CD4+ T cells. In phocytosis, and aff ected organs may also demonstrate addition, infected cells are targeted by cytotoxic T lym- infi ltration with cytotoxic T cells. Generalized lymph- phocytes, and by antibody. It is less cytopathic toward adenopathy may persist during this phase. macrophages and dendritic cells. As immunodefi ciency progresses, opportunistic infection • Viral replication continues in lymphoid tissue at varying begins to appear. Early such infections include: rates, and a steady decline is seen in CD4+ lymphocyte • recurrent or persistent herpes simplex numbers and immune function over years, with conse- • herpes zoster quent increase in plasma viral load unless eff ective treat- ment is instigated. • As immune function declines, patients become suscep- Box 16-24 tible to a range of infective, autoimmune and neoplastic complications. Recommended baseline investigations for the newly Clinical features and diagnosis diagnosed HIV patient Acute HIV infection • CD4+ T cell count • More than 50%, and possibly up to 90%, of patients • HIV viral load will suff er an acute (seroconversion) illness from 3 to 6 • HIV resistance assay (genotypic/phenotypic, weeks aft er being infected with the virus. Clinical fea- depending on local availability) tures are outlined in Box 16- 23. • Full blood count • Electrolytes, urea, creatinine Box 16-23 • Liver function tests Clinical features of acute HIV • Fasting lipids and glucose infection • Urinalysis • Hepatitis B and C serology Common Less common • Toxoplasma serology • Fever • • Serum cryptococcal antigen • Generalized • Splenomegaly • CMV serology lymphadenopathy • Diarrhea • Screening for other STIs (syphilis, chlamydia, • Maculopapular rash • Arthralgia gonorrhea) • Headache • Night sweats • HLAB*5701 (to detect risk of abacavir hypersensitivity) • Pharyngitis • Productive cough CMV, cytomegalovirus; HIV, human immunodefi ciency virus; STI, • Myalgia • Meningitis sexually transmitted infection. • Constitutional Adapted from: http://aidsinfo.nih.gov/contentfi les/AdultARV_GL_ symptoms Table3.jpg

552 Chapter 16 Immunology

Box 16-25 Autoimmune diseases seen in HIV infection Immune thrombocytopenic purpura Polymyositis Lymphoid interstitial pneumonitis Bell’s palsy Sjögren’s syndrome Autoimmune thyroid disease Antiphospholipid syndrome Guillain–Barré syndrome

• oral hairy • oropharyngeal candidiasis. When immunodefi ciency becomes profound, generally coinciding with a CD4+ T lymphocyte count of <200/m3, life-threatening infections such as (Figure 16- 20) and malignancies such as Kaposi’s sarcoma (Figure 16-21) become frequent. Management Comprehensive management of the HIV-infected patient requires attention to the psychological, environmental, socioeconomic, sexual and physical factors aff ecting each individual. Access to medical treatment, specifi cally anti- Figure 16-20 Chest X-rays and corresponding CT retroviral therapy, is obviously a key consideration. scans of pneumocystis pneumonia in chronic HIV The complexities of such management are beyond the infection; note the interstitial changes progressing to scope of this text. The principles of pharmacotherapy, how- ground-glass appearance ever, are key to the successful treatment of the disease, as From Goldman L and Schafer AI. Goldman’s Cecil medicine, 24th outlined in Table 16- 17 (overleaf ). ed. Philadelphia: Elsevier, 2012. Combined antiretroviral therapy (cART) Aims of therapy • Control viral replication. • Maintain or restore immune function (specifi cally CD4+ T cells) and reduce HIV-associated morbidity. • Prevent emergence of resistance (at least three antiviral agents aimed at two molecular targets). • cART is also used for pre-exposure prophylaxis (PrEP) and post-exposure prophylaxis (PEP).

When to initiate therapy Current guidelines are: • cART is recommended for all infected individuals, regardless of CD4+ T cell count. • Commence with three eff ective drugs » A ‘backbone’ of two nucleos(t)ide reverse transcrip- tase inhibitors » An ‘anchor’ of a non-nucleotide reverse transcrip- tase inhibitor, a boosted protease inhibitor or an Figure 16-21 Cutaneous Kaposi’s sarcoma in an AIDS integrase inhibitor. patient • The earlier cART is initiated aft er infection, the bet- From Hoff man R et al. , basic principles and practice, ter the prognosis. However, this needs to be weighed 5th ed. Philadelphia: Elsevier, 2009.

553 Essentials of internal medicine

Table 16-17 AIDS-defi ning conditions and their treatment

CONDITION TREATMENT PRINCIPLES Infections Esophageal, bronchial, tracheal or pulmonary Azole antifungals candidiasis Pneumocystis jirovecii pneumonia High-dose oral or IV co-trimoxazole, or IV , plus

prednisolone 1 mg/kg/day for 21 days if PaO2 < 70 mmHg on room air Cerebral toxoplasmosis Pyrimethamine, plus sulfadiazine or Cytomegalovirus IV ganciclovir or foscarnet, or oral valganciclovir Longer duration of treatment required for CMV retinitis Cryptosporidiosis Restoration of immune function essential; paromomycin, azithromycin not proven to be eff ective , disseminated or extrapulmonary Amphotericin-B or azole antifungals Cryptococcosis, extrapulmonary IV amphotericin-B with fl ucytosine, followed by oral fl uconazole Herpes simplex bronchitis, pneumonitis, esophagitis, Acyclovir (), valacyclovir or famciclovir or ulcer lasting >1 month Histoplasmosis, disseminated or extrapulmonary Amphotericin-B or azole antifungals Chronic intestinal isosporiasis Co-trimoxazole Mycobacterium tuberculosis infection Standard four-drug therapy as fi rst-line: , isoniazid, pyrazinamide, ethambutol Multi- is found in many parts of the world, necessitating local protocols Interactions with ARVs are complex and must be considered Mycobacterium avium complex or Clarithromycin, ethambutol, rifabutin M. kansasii Recurrent salmonella septicemia As per standard treatment guidelines Recurrent bacterial pneumonia As per standard treatment guidelines Progressive multifocal leukoencephalopathy Combination ART Malignancy Kaposi’s sarcoma Control of HIV infection with ART; radiotherapy or combination chemotherapy in rare instances Cerebral lymphoma As per standard treatment guidelines Burkitt’s lymphoma As per standard treatment guidelines Immunoblastic lymphoma As per standard treatment guidelines Invasive cervical cancer As per standard treatment guidelines Other HIV-associated wasting (>10% body weight plus Control of HIV infection with ART diarrhea, weakness and fever) HIV-associated dementia Control of HIV infection with ART

ART, antiretroviral therapy; ARV, antiretroviral drug; HIV, human immunodefi ciency virus.

554 Chapter 16 Immunology

against the patient’s preferences and psychosocial con- • patients who have experienced treatment failure; non- text, and the cost-eff ectiveness of early treatment. adherence is a major risk factor for this • Eff ective treatment with cART can reduce the risk of • all classes of ARV. HIV infection by any mode of transmission. It should be tested for in: • therapy- näive patients CLINICAL PEARL • patients who have experienced treatment failure, to The institution of antiretroviral therapy (ART) carries the guide treatment changes. risk of triggering immune reconstitution disease (IRD). In the virally-suppressed patient on long-term HIV treat- The risk is higher in patients who are more immuno- ment, it is important to screen for and manage potential suppressed at initiation of ART (lower CD4 count), comorbidities: and those who have existing opportunistic infection, even if treated. In IRD, the recovering immune system • hyperlipidemia triggers a dysregulated, often exuberant, infl amma- • hypertension tory response to the existing opportunistic infection. • glucose intolerance and diabetes mellitus This can result in clinical fl ares of disease that carry a high risk of mortality, especially with cryptococcal and • proteinuria mycobacterial infections. A range of autoimmune phe- • osteoporosis. nomena can also occur. Patients with IRD may require corticosteroid therapy. • depression and other psychosocial issues. Prognosis Factors impairing effi cacy of therapy • Prognosis has improved dramatically in all areas of the • Non- adherence. world where treatment with ARVs is available. • Poor tolerability. • Prognosis is adversely aff ected by socioeconomic fac- • Drug interactions between antivirals, or other medica- tors, and concurrent morbidities, especially pulmonary tions, that reduce eff ective drug levels. tuberculosis in the developing world. Toxicities of antitretroviral drugs (ARVs) are outlined in • Non-AIDS events are now the major cause of death in Table 16- 18. patients receiving treatment, and the management of HIV in an aging population is a new concern. Drug resistance • While modern ARVs have improved safety profi les, the Drug resistance has been documented in: challenge now is to further minimise drug toxicity and to manage each patient’s comorbid risk factors. • therapy-näive patients who have been infected with resistant virus

Table 16-18 Antiretroviral drugs

IMPORTANT SPECIFIC DRUG DRUG TARGET DRUG EXAMPLES TOXICITIES

Preventing viral entry—chemokine Maraviroc Hepatotoxicity receptor antagonists Rash Pyrexia Upper respiratory tract infection Cough GI intolerance

Preventing viral entry—fusion inhibitors; Enfuvirtide (T20) Injection site reactions prevent gp41 from achieving fusion-active GI intolerance conformation Fatigue

Preventing reverse transcription— Abacavir Abacavir—hypersensitivity reaction nucleoside reverse transcriptase Emtricitabine in those with HLA-B*5701; rash; inhibitors; analogues of native nucleoside increased cardiovascular risk Lamivudine substrates Zidovudine—bone-marrow suppression Zalcitabine

continues

555 Essentials of internal medicine

Table 16-18 Antiretroviral drugs continued

IMPORTANT SPECIFIC DRUG DRUG TARGET DRUG EXAMPLES TOXICITIES

Preventing reverse transcription— Tenofovir disoproxil fumarate (TDF) TDF—renal impairment, reduced bone nucleotide reverse transcriptase inhibitors Tenofovir alafenamide (TAF) density

Preventing reverse transcription— Etravirine Efavirenz—neuropsychiatric changes; non-nucleoside reverse transcriptase Delavirdine nightmares; teratogenic inhibitors binding to reverse transcriptase, Nevirapine—rash; hepatotoxicity preventing its action Efavirenz Nevirapine Rilpivirine

Preventing integration of HIV into host Elvitegravir Raltegravir—rhabdomyolysis cell genome—integrase inhibitors Raltegravir Dolutegravir—neuropsychiatric Dolutegravir changes Bictegravir

Preventing viral polypeptide splicing— Atazanavir Class toxicities—hyperlipidemia; protease inhibitors Fosamprenavir hepatotoxicity Darunavir Atazanavir—hyperbilirubinemia; prolonged PR interval; nephrolithiasis Indinavir Lopinavir—diabetes Saquinavir Ritonavir (used to inhibit metabolism and enhance blood levels of other protease inhibitors)

GI, gastrointestinal.

556 Chapter 16 Immunology

SELF-ASSESSMENT QUESTIONS

1 A 16-year-old male has perennial symptoms of nasal blockage, excessive sneezing and ocular itch. Which of the following statements is most likely to be TRUE for this patient? A Skin-prick allergy tests will be strongly positive for birch and ryegrass. B Allergen avoidance has no role in the management of his condition. C A cow’s-milk-free diet may improve his symptoms. D Positive serum-specifi c IgE for Dermatophagoides pteronyssinus probably explains the cause of his problem. E Eff ective treatment of this problem is unlikely to improve the management of his coexistent bronchial asthma. 2 A 44-year-old female presents with daily episodes of urticaria for approximately 6 months. The urticarial lesions are occurring in a generalized distribution. There have also been four episodes of moderately severe angioedema, aff ecting the lips and periorbital tissues. The patient has not noted any physical factors that provoke the problem. Which of the following actions would be most appropriate in this circumstance? A Organize for skin-prick allergy tests to a broad range of foods and . B Prescribe fexofenadine 180 mg twice daily for a minimum period of 2 months. C Prescribe a short course of prednisolone 50 mg daily to achieve control of the problem. D Place the patient on a modifi ed diet that excludes common allergenic foods. E Order testing for complement components C4 and C1q to exclude hereditary or acquired angioedema. 3 A 50-year-old man suff ers acute hypotension during an operation to remove his gallbladder under general anesthesia. The anesthetist notes some urticarial lesions on the torso and increased airway pressures, and treats the patient with epinephrine and fl uid volume expansion with good eff ect. Which two of the following actions would constitute appropriate follow-up? A Take blood immediately for serum tryptase measurement. B Organize for skin-prick and intradermal testing to the anesthetic drugs used during the procedure as an outpatient. C Strictly avoid all drugs used during the procedure in future. D Prescribe the patient an epinephrine self-injecting device. E Perform a skin test with the agents being used for future anesthesia in the anesthetic bay prior to any further anesthesia, to ensure no new allergies have developed. 4 A 25-year-old male presents with central chest pain of subacute onset, made worse by inspiration and somewhat relieved by leaning forward. He has noticed a rash over his cheeks in recent weeks, and has been feeling fatigued. His wrists and fi ngers have been aching, especially in the mornings. Physical examination reveals a malar rash with sparing of the nasolabial folds. There are no abnormal cardiac signs, and no obvious joint swelling. Dipstick urinalysis is negative for blood and protein. Laboratory investigations reveal erythrocyte sedimentation rate 45 mm/h (reference range [RR] 1–35 mm/h), C-reactive protein 4.8 mg/L (RR 0.0–5.0 mg/L), antinuclear antibodies 1:640 (homogeneous pattern), double-stranded DNA antibody negative, and microscopic urine examination normal. Which of the following statements is correct in this case? A The patient should be commenced on prednisolone at a dose of 1 mg/kg/day. B The chest pain is unlikely to be related to the other symptoms. C Hydroxychloroquine at a dose of 200 mg twice daily should relieve the symptoms within 48 hours. D Lack of a pericardial rub makes pericarditis unlikely. E An electrocardiogram, full blood count and extractable nuclear antigen (ENA) test would be appropriate in this situation. 5 A 75-year-old female with a 50-year history of smoking 30 cigarettes per day presents with severe headaches for 2 weeks. She has been taking prednisolone 5 mg daily for 4 months for the diagnosis of polymyalgia rheumatica (PMR). She denies ocular symptoms or night sweats. She has recently lost several kilograms in weight. She has a longstanding cough with morning sputum production, which has worsened over the past few months. Physical examination reveals a thin woman with a temperature of 37.8°C. There is mild bilateral scalp tenderness but no obvious thickening of the temporal arteries. Which of the following approaches would be appropriate in this situation? A Arrange for chest radiograph and head computed tomography (CT) for the presumptive diagnosis of lung cancer with cerebral metastases. B Organize an urgent temporal artery biopsy to exclude giant-cell arteritis (GCA). C Increase the prednisolone dose to 50 mg daily and organize an urgent temporal artery biopsy. D Arrange urgent ophthalmological review to exclude ophthalmic artery vasculitis. E Arrange for urgent erythrocyte sedimentation rate (ESR) and full blood count, and only increase the steroid dose if these have shown substantial change from previous levels.

Questions continue overleaf.

557 Essentials of internal medicine

6 A 60-year-old female patient develops pallor and pain of the right hand while camping in cold weather, which last for about an hour. She subsequently develops a persistent dry cough, mild dyspnea on exertion, and feels ‘weak’. Physical examination reveals bilateral basal inspiratory lung crackles, no signs of pulmonary hypertension, and marked symmetrical weakness of proximal muscle groups. Which combination of abnormal laboratory results is most likely to correspond to this clinical situation? A Antinuclear antibodies positive 1:1280, anti-Jo-1 antibodies positive, CK (creatine kinase) 853 U/L (reference range [RR] 26–140 U/L) B Antinuclear antibodies positive 1:320, anti-RNP antibodies positive, CK 299 U/L (RR 26–140 U/L) C Antinuclear antibodies positive 1:2560, anti-SS-A antibodies positive, anti-SS-B antibodies positive, rheumatoid factor 42 IU/mL (RR 0–14 IU/mL), CK 130 U/L (RR 26–140 U/L) D Antinuclear antibodies negative, rheumatoid factor 102 (RR 0–14 IU/mL), CK 53 U/L (RR 26–140 U/L) E Antinuclear antibodies positive 1:2560, anti-ds-DNA 27 IU/mL (RR 0–6 IU/mL), anti-Sm antibodies positive, CK 330 U/L (RR 26–140 U/L) 7 A 52-year-old non-smoking male presents with a left-hemisphere ischemic stroke. Subsequent investigations reveal:

Activated partial thromboplastin time (APTT) 37 seconds Reference range (RR) 25–39 seconds

Dilute Russell viper venom time (DRVVT) and phospholipid neutralization:

Patient DRVVT 40 seconds RR 32–42 seconds

DRVVT + phospholipid 36 seconds

DRVVT (normalized) ratio 1.0 seconds <1.2 = negative 1.2–1.5 = weak positive 1.6–2.0 = moderate positive >2.0 = strong positive

Kaolin ratio 1.1 (RR <1.2)

Cardiolipin IgG antibodies 75 GPL (RR <5 GPL)

Cardiolipin IgM Abs 45 MPL (RR <5 MPL)

Beta-2-glycoprotein 1 Abs 33 SGU (RR <20 SGU)

GPL, unit equivalent to 1 microg of ; MPL, unit equivalent to 1 microg of ; SGU, standard IgG anti-beta glycoprotein 2 international units.

Which of the following management approaches would be most appropriate in this situation? A Aspirin 100 mg daily long-term. B Subcutaneous therapeutic-dose enoxaparin for 3 months, then recheck cardiolipin antibody levels. If levels remain positive, institute lifelong warfarin therapy. C Therapeutic-dose intravenous unfractionated heparin until warfarin therapy has been instituted to therapeutic levels. Continue warfarin therapy lifelong. D Standard therapy for stroke. Recheck cardiolipin antibodies in 3 months. If the IgG cardiolipin antibody level remains positive, institute lifelong warfarin therapy. E Immediately commence aspirin, intravenous heparin, and warfarin. Once therapeutically warfarinized, discontinue heparin but continue aspirin and warfarin 8 A 21-year-old woman has had fi ve signifi cant episodes of sinusitis and three of pneumonia in the previous 12 months. Which of the following pathology results would be consistent with the most likely diagnosis in this case? A Normal total IgG levels, but reduced IgG2 and IgG4 B Reduced total hemolytic complement level C Persistent reduction in the absolute neutrophil count D Reduced levels of blood CD4+ lymphocytes on fl ow cytometry E TACI mutation 9 A 35-year-old man with stable human immunodefi ciency (HIV) infection for 5 years on treatment with two nucleoside reverse transcriptase inhibitors and efavirenz is found to have detectable HIV viremia (at a low level) for the fi rst time since commencing antiretroviral therapy. Which of the following would be the most appropriate course of action? A Perform antiretroviral resistance testing, and add a protease inhibitor. B Suspend antiretroviral therapy while waiting for antiretroviral resistance testing and CD4+ T-cell numbers. C Perform antiretroviral resistance testing and change to a diff erent three-drug regimen. D Perform antiretroviral resistance testing, CD4+ T-cell numbers, and maintain the current regimen pending the results. E Review and reinforce medication adherence, and arrange to re-examine HIV viral load at the next clinical review.

558 Chapter 16 Immunology

ANSWERS

1 D. This is a typical case of persistent allergic rhinoconjunctivitis, which is usually caused by exposure to perennial allergens such as house dust mite or animal danders. The demonstration of specifi c IgE to Dermatophagoides pteronyssinus strongly suggests that the house dust mite is the cause. Specifi c house dust mite reduction measures, particularly in the bedroom, may help alleviate symptoms, and should be employed in addition to other therapy. Cow’s milk does not cause persistent allergic rhinoconjunctivitis. Birch or ryegrass allergy causes seasonal (spring and summer) symptoms rather than perennial ones. There is evidence that improved control of allergic rhinitis symptoms results in improved asthma control.

2 B.

Of the options given, prescription of a prolonged course of adequate-dose non-sedating H1 antihistamines is the appropriate course of action in chronic idiopathic urticaria and angioedema (CIU/A). CIU/A is not an allergic or diet-related condition, so allergy testing and elimination diets are not appropriate. Corticosteroids should be avoided in CIU/A if at all possible, as this is a chronic problem, and reliance on steroids is likely to cause signifi cant morbidity. The exception would be if a severe bout of angioedema was present. Angioedema secondary to a complement defi ciency is not associated with urticaria. Failure of high-dose antihistamine therapy to control the symptoms should prompt consideration of the use of omalizumab.

3 A and B. This is a typical history of anaphylaxis to general anesthesia. Measuring serum tryptase promptly will help confi rm the diagnosis of anaphylaxis. Formal skin testing in a specialized clinic for allergy to the agents used—and generally to a standard panel of anesthetic drugs—is essential to identify the culprit. This will also allow the patient to have access to the drugs which were not the cause of the reaction in the future. It may be very diffi cult to avoid all of the drugs which were administered on this occasion in the future if the patient requires further surgery. The patient does not need an epinephrine self-injecting device if his only anaphylaxis risk is general anesthesia! Non-standardized skin testing is not a safe way to diagnose drug allergy.

4 E. This patient most likely has acute pericarditis in the setting of systemic lupus erythematosus (SLE). A combination of malar rash, arthralgia and positive antinuclear antibody in a young male has a high positive predictive value for the disease. Elevated erythocyte sediment rate (ESR) with normal C-reactive protein is typical of SLE. Pericardial rub is an insensitive sign for pericarditis, so an electrocardiogram and echocardiogram should be performed. The presence of anti-Sm, anti- SS-A, or anti-SS-B on ENA testing would be strong evidence of SLE, although sensitivity is low. Evidence of a lymphopenia or normochromic, normocytic anemia would also be consistent with SLE. Treatment in this situation would be appropriate with non-steroidal anti-infl ammatories, and possibly low-dose corticosteroids. There is no apparent indication for high- dose steroids. Hydroxychloroquine is a good choice for ongoing therapy, but its eff ect is not rapid so other treatment is needed in the short term.

5 C. Approximately 20% of patients with PMR will go on to develop GCA. Being on low-dose corticosteroids may disguise some of the symptoms and signs of GCA, so the lack of visual symptoms or palpable temporal arteries should not reassure the clinician that GCA has not developed. Similarly, the lack of serological evidence of an acute phase response, while being unusual, does not exclude GCA altogether. Headache is the most common symptom of GCA. To miss the diagnosis of GCA and not treat it, only to have a patient go blind or have a stroke, is a disaster. The approach should always be to treat the patient for GCA while organizing diagnostic tests. While involvement of an ophthalmologist in the case may be important, they will not usually be able to diagnose changes due to GCA in the absence of ocular symptoms.

6 A. The combination of Raynaud’s phenomenon, myopathy and probable interstitial lung disease is a classic presentation of the anti-synthetase variant of polymyositis. The presence of antinuclear antibodies and anti-Jo-1 antibodies would be consistent with this, and the markedly elevated CK level would indicate signifi cant myositis. Answer B would be more consistent with mixed connective tissue disease, where pulmonary fi brosis would be unlikely. Answer C is likely to be Sjögren’s syndrome. The fi ndings in answer D could be present in rheumatoid arthritis, where one would not expect a myopathy. Answer E is consistent with systemic lupus erythematosus, where myopathy to this degree and rapidly developing pulmonary fi brosis would be unusual. Table 16-19 summarizes disease associations with autoantibodies.

559 Essentials of internal medicine

Table 16-19 Autoantibodies and disease associations

DISEASE AUTOANTIBODY PREVALENCE/COMMENT Systemic lupus ANA >95%; homogeneous pattern more specifi c erythematosus (SLE) ds-DNA ~50%; high specifi city; risk factor for renal disease SS-A 30–50%; also found in Sjögren’s syndrome SS-B 10%; also found in Sjögren’s syndrome Sm 20–30%; high specifi city Drug-induced lupus Histone >70% specifi city; not associated with drug-induced lupus due to anti-TNF (tumor necrosis factor) therapy (infl iximab, etanercept) Sjögren’s syndrome ANA Up to 80%; speckled pattern associated with SS-A and SS-B

SS-A (Ro) 60–95%; responsible for neonatal lupus (only about 2% of SS-A-positive women will have babies with neonatal lupus) SS-B (La) 40–90%

Polymyositis (PM)/ ANA 60–90%; variable pattern dermatomyositis (DM) Myositis-specifi c antibodies: 20%; associated with anti-synthetase syndrome; more Anti-synthetase including Jo-1 common in PM than DM Anti-signal recognition Not routinely measured Anti-Mi-2 Not routinely measured Myositis-associated Not routinely measured antibodies: includes anti-PM- Scl, anti-U1RNP, anti-Ku Scleroderma ANA Up to 90%; nucleolar pattern reasonably specifi c Scl-70 ~50% Centromere ~10% CREST syndrome ANA Up to 90%; nucleolar pattern reasonably specifi c Centromere ~80% Scl-70 <10% Mixed connective ANA >90% tissue disease RNP Approaches 100% sensitivity; also found in SLE and scleroderma Antiphospholipid Cardiolipin Demonstration of one of these antiphospholipid antibodies syndrome (APS) Lupus inhibitor necessary for diagnosis of APS; 25% of SLE patients are

Beta2-GP-1 antibody-positive Granulomatosis with c-ANCA (proteinase 3) >90% polyangiitis (Wegener’s granulomatosis) p-ANCA (myeloperoxidase) Occasional

Microscopic p-ANCA (myeloperoxidase) 80% polyangiitis c-ANCA (proteinase 3) Occasional Eosinophilic p-ANCA (myeloperoxidase) 50% granulomatosis with polyangiitis (Churg– c-ANCA (proteinase 3) Occasional Strauss syndrome)

560 Chapter 16 Immunology

7 C. The results show strongly positive anti-cardiolipin and beta-2-glycoprotein 1 antibodies, consistent with the anti- phospholipid syndrome (APS), and very likely to have caused a thromboembolic stroke. The lupus inhibitor is not present, as evidenced by the normal DRVVT and phospholipid neutralization, and normal APTT. Russell viper venom (RVV) contains a coagulant that activates factor X, which in turn converts prothrombin into thrombin in the presence of phospholipid and factor V. The DRVVT will therefore be prolonged in the case of either the presence of an inhibitor of phospholipid or a defi ciency of factor V or prothrombin. In the laboratory, a standard sample of RVV and phospholipid is used that will give a clotting time within a range when combined with normal serum. Abnormal serum that contains some types of antiphospholipid antibody will result in a prolonged time. The lupus inhibitor prolongs the DRVVT, whereas most other anti-cardiolipin and beta-2-glycoprotein 1 antibodies do not. Without anti-coagulation, this patient is at high risk of further thrombosis. Antiplatelet therapy is not adequate therapy, and there is no evidence that both anticoagulation and antiplatelet therapy need to be used concurrently in APS. Technically, the presence of the anti-cardiolipin antibodies needs to be documented on two occasions 12 weeks apart for the diagnosis of APS, but there is no reason to use enoxaparin for 3 months rather than warfarin. In the unlikely event that the anti-cardiolipin antibody levels are no longer elevated when checked at 3 months, and if no other predisposing factors to stroke have been identifi ed in this patient, it would still be advisable to continue lifelong anticoagulation.

8 E. A mutation in TACI is found in a small number of patients with common variable immunodefi ciency (CVID). This case history is typical of CVID, which tends to come on in teenagers or young adults, and classically results in recurrent respiratory tract infection. IgG subclass defi ciency has not been shown to signifi cantly increase the risk of recurrent infection. Complement defi ciencies usually present with invasive infection due to encapsulated organisms, such as Neisseria meningitides or . Neutropenic disorders are more likely to present with abscesses, septicemia, and oral mucosal infections. Reduced levels of T cells are associated with opportunistic viral and fungal infections.

9 D. In this situation, the most likely explanation for the loss of viremic control is non-adherence to therapy. There are multiple possible explanations for this: adverse drug reactions, concern over long-term medication use, psychosocial factors, or simple diffi culty with remembering to take daily medication. It is important to explore these issues with the patient and use this opportunity to reinforce the importance of adherence. Drug resistance is very possible if adherence has been patchy; however, testing may not be able to be performed if there is low level viraemia only. Any change in treatment, however, is best made with knowledge of the resistance pattern.

561